MCAT questions wrong chemistry

Ace your homework & exams now with Quizwiz!

∆Hf

"heat of formation". The energy needed to build one mole of substance from raw elements.

inert gas

(also, noble gas) element with filled outer electron shell that is unreactive with other atoms

Robinson Annulation Reaction

*Robinson annulation* is a two-step method for forming a 6 membered C ring, in which a Michael addition is followed by an intramolecular aldol condensation produces alpha-beta unsaturated ketones

H oxidation number

+1 with nonmetals, -1 with metals

Hydrogen oxidation number

+1 with nonmetals, -1 with metals

oxidation number of Fe+2

+2, in monatomic ions, oxidation number of equal to charge

OH - oxidation number

-1

halogen oxidation number

-1 normally unless bonded to a more electronegative atom or O, N

Oxygen oxidation number

-2 (except peroxides)

which is more acidic -OH or -SH

-SH ARIO A-atom S is larger than O therefore it can spread it negative charge over a larger area

pKa

-log(Ka) SMALLER pKa, STRONGER acid

equation to find pKa from Ka

-logKa

if a compound has both an OH and NH2 in it, will the suffix be -ol or -anamine? where does the amine get named?

-ol because the alcohol is considered more important the amine will act like a normal substituent where its location is described by the number carbon its attached to and it is just called amine

Pure element oxidation number

0

cos 90 =

0

ln(1)=

0

What is the concentration of Cl- ions in a 0.1 M solution of calcium chloride? A. 0.02 M B. 0.05 M C. 0.10 M D. 0.20 M

0.20 M after dissociation in water there will be 1 Ca ion for every 2 Cl ions for 0.1 M of CaCl2 there will be 2 Cl ions so 0.1 M is multiplied by 2 to do 0.2 M

cos 0°=

1

sin(90)=?

1

Calorie vs calorie

1 Calorie = 1000 calories = 1 kcal

What is the approximate molarity of sodium chloride in ocean water, if the density of ocean water is 1.028 kg/L? if ocean water is 3.5% is salt

1 L of ocean water has a mass of 1028 g, of which 3.5% is salt. Note that the molar mass of NaCl is approximately 58 g. (1028 g)(0.035) = 36 g NaCl (36 g NaCl) / (58 g NaCl/mol) = 0.62 moles 0.62 moles / 1 L = 0.62 M

Hypo-_____-ite

1 less oxygen than -ite

what is the name of this molecule

1-hexanol or hexanol alcohols have the suffix -ol

What does ARIO stand for?

1. Atom- electronegative atoms like O are the best at handling a negative charge 2. Resonance- the resonance of a negative charge adds stability especially when both atoms are electronegative 3. Induction- electronegative atoms that pull some electrons away from the negatively charged base 4. Orbital- where the charge resides sp > sp2 > sp3, sp is most stable the order that predicts acidity from largest effect to smallest

Balancing Redox Reactions

1. Separate the two half reactions 2. Balance the atoms of each half reaction(may have to add H2O or H) 3.Balance the electrons of each half reaction 4.Combine the half reactions 5.Confirm mass and charge are equal

electron energy equation

1/wavelength is Energy

enantiomeric excess equation if there is 10º observed rotation and 20º rotation of enantiomer S what percent of the solution is S and what percent is R

10/20 = .5 x 100 = 50% 50% excess of S so 75% S 25% R

boiling point of water

100ºC

hydronium ion concentration from pH

10^-pH

pKa of alcohol

16 (not much different from water)

Nomenclature Prefix's for Ochem compounds, 1-10

1: none 2: eth 3: prop 4: but 5: pent 6: hex 7: hept 8: oct 9: non 10: dec

n=1 subshell n=2 sublevels n=3 sublevels n=4 sublevels

1s 2s, 2p 3s, 3p, 3d 4s, 4p, 4d, 4f

What is the name of this structure?

2-heptanamine molecules with NH2 have the suffix -anamine

rank in order of increasing boiling point A. hexane B. butane C. 2-methylpropane

2-methylpropane butane hexane longer carbon chains have higher boiling points because they have more bonds to break and branched alkanes have slightly lower boiling points because they stack and interact together less frequently

at STP, 1 mole of gas takes up ____ L

22.4 L of volume

Standard conditions

273K(0 ºC) and 100kPa (1 atm)

speed of light

3.00 x 10^8 m/s

C3H8 + 5O2 = what type of reaction is this

3CO2 + 4H2O + heat combustion

what is this molecules name and why is it numbed the way it is

4-ethyl-5-isopropyl-3,3-dimethyloctane the numbers are in this direction and not the other way so the substituents have the smallest number possible

Huckle's Rule of Aromaticity

4n+2 = x find X by counting and then solve for n if n is a whole number then the molecule is aromatic and very stable

what is the pH of an unknown aqueous solution with a hydrogen ion concentration of 1 × 10^-5 M 4 5 9 10

5 pH of solution corresponds to the power of the hydrogen concentration works similarly with basic solutions as well A solution with pH 10 has a hydronium ion concentration of 1 × 10^−10

What is a racemic mixture?

50/50 mixture of two enantiomers

Avogadro's number and definition

6.02 x 10^23 represents the number of particles in a mole

Plank's constant

6.626 x 10^-34 a number used to calculate the radiant energy (E) absorbed or emitted by a body based on the frequency of radiation

The Doppler effect states: A. The speed of sound B. The change in the wavelength of a sound C. A change in sound frequency caused by motion of the sound source D. Musical sound is perceived by the ear as a set of a number of constituent pure harmonic tones.

A change in sound frequency caused by motion of the sound source, motion of the listener, or both.

alpha particle

A cluster of 2 protons and 2 neutrons emitted from a nucleus in one type of radioactivity Positively charged

meso compound

A compound with chirality centers and an internal plane of symmetry causing it to be an achiral molecule

which molecule in the photo is a meso compound

A meso compound is a stereoisomer with an internal plane of symmetry the left one

Chemiosmosis

A process for synthesizing ATP using the energy of an electrochemical gradient and the ATP synthase enzyme.

HA + H2O --> ?

A- + H3O+ makes your proton H3O+ as seen in acidic solutions

What is the approximate pH of a saturated aqueous solution of hydrochloric acid whose molarity is 10.6 M? A. -1 B. 1 C. 7 D. 13

A. -1 Hydrochloric acid is a strong acid and completely dissociates in aqueous solution. In this solution, the hydronium ion concentration is 10.6 M, which can be approximated as 10 M to make the math easier. The pH is the -log of the hydronium ion concentration: -log[10] = -log[10^1] = -1. While the typical pH range is normally thought of as ranging from 0 to 14, if the concentration of hydronium ion is greater than 1 M, negative pH values are possible. It is also possible to have pH values greater than 14, i.e. if the hydroxide concentration is greater than 1 M.

When 2 moles of hydrofluoric acid are added to 100 mL of water, the resulting solution has a pH equal to 4. What is the percent dissociation of HF? A. 0.0005% B. 0.05% C. 0.5% D. 5%

A. 0.0005% Remember that HF is a weak acid, meaning that it does not fully dissociate in water. In many weak acid problems, you must use an ICE table and the Ka of the acid to calculate the pH of an acidic solution. However, this question gives us pH outright. Remember, pH = -log[H+], so [H+] = 10^-pH. Here, [H+] = 10^-4 M. Now, we must understand what is meant by "percent dissociation." The percent dissociation of HF is simply the percent of the original acid concentration that has dissociated into H+ and F- ions. This value is equal to [H+]/[HF] x 100%. Original [HF] = 2 moles / 0.100 L = 20 M [H+]/[HF] = (10^-4 M H+) / (20 M HF) = 5 x 10^-6 % dissociation = 5 x 10^-6 x 100% = 5 x 10^-4 % dissociation = 5 x 10^-4% = 0.0005%

What is the ratio of hydronium ion concentrations in solution at the pH that results in the highest MP activity, 6, to that which results in the lowest MP activity, 4? A. 0.01 B. 0.1 C. 10 D. 100

A. 0.01 The answer is A because the highest activity occurs at pH 6 and the lowest activity occurs at pH 4. The difference of two pH units represents a 100 times higher concentration of hydronium ion at the lower pH. Therefore, the ratio is 1/100 or 0.01. This is a Scientific Reasoning and Problem Solving questions because you must recall the correct formula to solve the problem. [H+] = 10^-pH 10^-6/10^-4

If a modern portable defibrillator uses as 12 V battery and a 20 μF capacitor, what is the total charge stored on the plates of the capacitor? A. 0.24 mC B. 24 mC C. 24 C D. 60 C

A. 0.24 mC Capacitance (C, measured in farads) is the amount of charge stored per volt, expressed with the equation C = q/V. This means that charge (q) = VC. (Do you remember the home shopping network, QVC?) The capacitance is given as 20 x 10-6 F. Substituting the voltage (12 V) and capacitance into the equation gives us the charge (in coulombs). (20 x 10-6 F)(1.2 x 101 V) = 24 x 10-5 C = 2.4 x 10-4 C = 0.24 x 10-3 C = 0.24 mC Remember when making the exponent larger, we must make the coefficient smaller by the same factor.

Adenine contains (select all) A. 1 H donor B. 1 H acceptor C. 2 H donors D. 2 H acceptors

A. 1 H donor B. 1 H acceptor

thymine contains (select all) A. 1 H donor B. 1 H acceptor C. 2 H donors D. 2 H acceptors

A. 1 H donor B. 1 H acceptor

cytosine contains (select all) A. 1 H donor B. 1 H acceptor C. 2 H donors D. 2 H acceptors

A. 1 H donor D. 2 H acceptors

4.168 J = A. 1 calorie B. 1 Calorie

A. 1 calorie small calorie not the food calorie amount of energy needed to raise 1g of water by 1ºC

C=O on IR spectrum A. 1650 to 1750 B. 3200 to 3500 sharp peak C. 3200 to 3500 broad peak D. 2500 to 3300 and 1750

A. 1650 to 1750

Beta minus decay emits a beta- particle that has (select all) A. 1e B. 2e C. 1p D. 2p

A. 1e

What is the pH of a 0.010 M perchloric acid solution? A. 2 B. 4 C. 7 D. 12

A. 2 Perchloric acid is a strong acid that completely dissociates in aqueous solution, so the hydrogen ion concentration is 1.0 x 10-2 M. The pH = -log[H+] = -log[10^-2] = 2. Since Log[1Ex] = x

At what pH is histidine mainly charged A. 6 B. 7 C. 8

A. 6 His has a side chain pka of 6 this is where pka > pH therefore protonated at pH of 7 pH > pka therefore deprotonated and neutral

A pure sample of (R)-limonene has a specific rotation of +125.6. If a mixture of (R)-limonene and (S)-limonene has a specific rotation of +62.8, what are the percentages of the R and S enantiomers in this mixture? A. 75% R, 25% S B. 55% R, 45% S C. 45% R, 55% S D. 25% R, 75% S

A. 75% R, 25% S Since the enantiomers in this case must be present in unequal proportions, a weighted average of the components should be used. In this case, the weighted average is given; therefore, we can set up an equation. If we designate x as the percentage of R and 1 - x as the percentage of S, we can solve for x using this equation: (125.6)(x) + (-125.6)(1-x) = 62.8 125.6x - 125.6 + 125.6x = 62.8 251.2x = 188.4 x = 0.75 1 - x = 0.25 Therefore, R makes up 75% of the solution, while S comprises the remaining 25%.

the equivalence point of a strong acid/weak base titration is A. < 7 B. 7 C. > 7

A. < 7 the products are H2O, salt, and the conjugate acid H2O and salt are neutral the conjugate acid of weak bases is stronger so it has an effect on pH

What is the correct definition of a meso compound A. A compound with chirality centers and an internal plane of symmetry causing it to be an achiral molecule B. rotated molecule that can be superimposed on its mirror image C. subtype of diastereomers that differ in absolute configuration at exactly one chiral carbon

A. A compound with chirality centers and an internal plane of symmetry causing it to be an achiral molecule

The rates of diffusion of four drugs were tested: acetazolamide (pKa = 7.2), sulfadiazine (pKa = 6.5), warfarin (pKa = 5.0), and cephalexin (pKa = 3.6). Which drug will have the strongest conjugate base? A. Acetazolamide B. Sulfadiazine C. Warfarin D. Cephalexin

A. Acetazolamide The Ka and pKa values of acids are inversely related to each other (pKa = -log [Ka]) Thus, the highest pka will have the lowest Ka (Ka = [H+][A-]/[HA]) The highest Ka (lowest pka) will have the most dissociation meaning it is the strongest acid the lowest Ka (highest pka) will have less dissociation and be a weaker acid, weaker acids have stronger conjugate bases.

Following radioisotope-aided imaging, the isotope 99Tc decays to 99Ru and is cleared by the kidneys. What particle is ejected during this additional decay step? A. An electron B. A positron C. A gamma photon D. An alpha particle

A. An electron this is beta - decay because a neutron became a proton this would emit an electron

Which of the bromomethanes is LEAST polar? A. CBr4 B. CHBr3 C. CH2Br2 D. CH3Br

A. CBr4 The polarity of each of the bromomethanes is determined by the vector sum of all of the individual bond polarities within a given molecule of the compound. Carbon is sp3 hybridized and tetrahedral in all of the bromomethanes. Therefore, the vector sum of the bond polarities in CBr4 is zero, whereas the vector sum is greater than zero in the other compounds. Thus, answer choice A is the best answer.

Which of the following molecules will have the highest Rf value when studied with thin-layer chromatography using a silica plate? A. CH3CH2CH2CH2CH3 B. HOCH2CH2CH2CH3 C. HOCH2CH2CH2OH D. HOOCCH2CH2CH3

A. CH3CH2CH2CH2CH3 Silica is a highly polar molecule; thus, when chromatography is done on a silica plate, a polar substance will move slower on the plate due to polar-polar interactions between the substance and the plate. This means that the Rf value of a polar substance will be smaller than that of a nonpolar substance since the substance will move less on the plate compared to the solvent front than a nonpolar substance would. Of the options given, the compound in answer choice A is the least polar. Remember, Rf is equal to the distance the compound of interest traveled along the plate divided by the distance traveled by the solvent front. A sample TLC plate, with those distances labeled, is shown below. Note that A, B, and C represent arbitrary compounds in this image.

Which of the following characteristic changes in the IR spectrum would indicate the conversion of a fatty acid to an ester? A. Disappearance of a broad peak in the 2500 to 3300 cm-1 region only B. Appearance of a broad peak in the 2500 to 3300 cm-1 region only C. Appearance of a broad peak in the 2500 to 3300 cm-1 region, disappearance of an intense sharp band in the range 1730-1750 cm-1 D. The disappearance of a broad peak in 2500 to 3300 cm-1 region, the disappearance of an intense sharp band in the range 1730-1750 cm-1

A. Disappearance of a broad peak in the 2500 to 3300 cm-1 region only In the conversion of a fatty acid to an ester, we lose a hydroxyl group, and neither gain nor lose any other relevant functional groups. Hydroxyl OH bonds in carboxylic acids sound in the range of 2500 to 3300 cm-1, making answer choice (A) the correct answer.

Select all that describe the molecule in the picture A. Enantiomers B. conformational isomers C. functional isomers D. Stereoisomers

A. Enantiomers mirror images of one another D. Stereoisomers only differ from each other by their stereochemistry all enantiomers are stereoisomers

When is ∆H positive? A. Endothermic B. Exothermic

A. Endothermic heat added to the system

Fischer esterification turns a carboxylic acid into a A. Ester B. alcohol C. hemiacetal

A. Ester

Which of the following properties of an ultrasound wave remains unchanged as it passes into human tissues? A. Frequency B. Wave speed C. Amplitude D. Wavelength

A. Frequency frequency of a wave is not affected by the medium through which it propagates.

which of the following are assumptions made for ideal gases? A. Gas particles have no volume B. Molecules always create 1 ATM on their container C. molecules have no forces between them D. molecules do not move

A. Gas particles have no volume C. molecules have no forces between them

Which NMR has splitting patterns? A. H1 B. C13

A. H1

Which of the following is a weak acid in aqueous solution? A. HF B. HCl C. HBr D. HI

A. HF All but one of the hydrohalic acids completely dissociate. The one exception is hydrofluoric acid, which is a weak acid in aqueous solution due to its unusually high degree of covalent bonding between the fluorine and the hydrogen.

The transcription factor AP-1 is a heterodimer consisting of c-jun and c-fos. C-jun and c-fos are soluble proteins that can be localized to either the cytosol or nucleus of a cell. C-jun and c-fos dimerize through a leucine zipper motif. In a leucine zipper motif, every 7 amino acid residues, or 2 full turns of an alpha helix, are leucine resides. Leucine and other amino acids on one face of the helix come together to form an opposite alpha helix that has a similar arrangement of leucine and other amino acids. Which solvent would be LEAST favorable for c-fos/c-jun dimerization? A. Hexane B. Ethanol C. Water D. Phosphate buffered saline

A. Hexane begin by summarizing exactly what the question is asking. In short, leucine residues on different parts of a molecule are coming together to form a dimer. Leucine is hydrophobic, since its side chain contains only carbon and hydrogen. If the solvent were also hydrophobic, the face of a leucine zipper could interact just as favorably with the solvent as with the opposite alpha helix. Some leucine residues would likely interact only with the solvent, preventing formation of the dimer entirely. Hexane, with its nonpolar hydrocarbon structure, is the least polar solvent listed.

Select all that apply: linear carbohydrates always contain a: A. Hydroxyl group B. Long hydrocarbon chain C. Carbonyl

A. Hydroxyl group C. Carbonyl

When experiments are performed on enzymes that display traditional Michaelis-Menten kinetics, what shape does the graph of V0 versus substrate concentration [S] have? A. Hyperbolic dependence on [S] B. Linear dependence on [S] C. Sigmoidal dependence on [S] D. Parabolic dependence on [S]

A. Hyperbolic dependence on [S]

Radioactive tritium (3H) labeled guanine has been used to measure the rate of biochemical processes that involve its binding or incorporation. (look at pic) Given that water is the solvent for this type of experiment, what is the best site for tritium labeling? A. I B. II C. III D. IV

A. I The answer to this question is A, because the best site for tritium labeling would not exchange the tritium ions for protons in water. All of the N−H sites (II−IV) would readily exchange tritium protons due to their lone pair-facilitating protonation and subsequent tritium exchange with water, but the C−H site (I), lacking a lone pair, would retain its tritium label.

what reactant can turn an aldehyde into carboxylic acid: select all that apply A. KMnO4 B. Strong oxidizing agent C. strong reducing reagent D. PCC E. K2Cr2O7

A. KMnO4 B. Strong oxidizing agent E. K2Cr2O7 PCC can turn alcohols into aldehydes but is not a strong enough oxidizing agent to turn it into a carboxylic acid

which conditions will have the highest catalytic efficiency A. Kcat: 8.7 ± 0.5, Km: 970 ± 220 B. Kcat: 6.7 ± 0.02, Km: 1900 ± 500 C. Kcat: 0.9 ± 0.02, Km: 140 ± 10

A. Kcat: 8.7 ± 0.5, Km: 970 ± 220 catalytic efficiency is Kcat/Km so we want the highest Kcat and lowest Km

Which of the following statements is NOT true concerning the element that it describes? A. Lithium has the second-largest atomic radius in period 2. B. Iodine has the highest electronegativity in period 5. C. Iron is a transition element. D. Neodymium is a rare earth element.

A. Lithium has the second-largest atomic radius in period 2. Lithium (atomic symbol Li) is in group 1, which is sometimes called the alkali metal group. This group has the largest atomic radii of the periodic table. Thus, lithium has the largest (not the second-largest) atomic radius in period 2, and choice A is the correct answer to this NOT question. Group 2 elements, such as magnesium and calcium, are known as the alkaline earth elements and would have the second-largest atomic radii.

When a solute is added to a solvent, the freezing point/melting point of the solution is A. Lower B. Higher C. the same

A. Lower solutes lower the freezing point by blocking the solvent particles from congregating why they add salt to roads

which are strong nucleophiles A. N3- B. CN- C. H2O- D. HO- E. R-OH

A. N3- B. CN- D. HO-

Turbulent flow in humans is a major risk factor for atherosclerosis, the buildup of plaque on arterial endothelium. Ignoring any potential effects of turbulence, what effect would atherosclerosis have on blood flow? A. Narrowing of the artery causes the velocity to increase and the hydrostatic pressure to decrease. B. Narrowing of the artery causes the velocity to increase and the osmotic pressure to decrease. C. Narrowing of the artery causes the velocity to decrease and the hydrostatic pressure to decrease. D. Expansion of the artery causes the velocity to decrease and the hydrostatic pressure to increase.

A. Narrowing of the artery causes the velocity to increase and the hydrostatic pressure to decrease. The buildup of plaque in the lumen of the vessel would cause a narrowing of the vessel. The question then implies that we must consider the Venturi effect, which states that a fluid's velocity must increase as it passes through a constriction, based on the continuity (A1v1 = A2v2) principle. According to the Bernoulli principle, then, the hydrostatic pressure (the pressure that any fluid in a confined space exerts, BP) must decrease to conserve energy.

Which of the following reagents would be involved in the first synthetic step to chemically derive an amide from oleic acid? A. Phosphorous pentachloride B. Aminobutane C. Octadecanol D. 2-aminopentane

A. Phosphorous pentachloride This question is a bit tricky. Remembering the hierarchy of relative reactivities of carboxylic acid derivatives is relatively easy, but applying the concepts under the time constraints of the MCAT is more challenging. Amides are the most stable (least reactive) of the carboxylic acid derivatives because the amino substituent is a very poor leaving group. In order to synthesize amides however, one cannot simply use the amino substituent as a nucleophile; as a relatively strong base, it will instead preferentially participate in an acid-base neutralization reaction with the acidic hydrogen of the organic acid. Instead, one must first create an acid chloride (or an acid anhydride, which is not an option in this question) to eliminate the confounding acidic hydrogen. Afterwards, one can proceed with a normal nucleophilic substitution. Aminobutane and 2-aminopentane are both suitable as amino substituents but are therefore unsuitable as a first step. The addition of octadecanol would be a poor choice of a first step; absent rigorous reaction conditions, it's unlikely anything productive would happen at all.

what is the molecular formula of an acyl halide A. RCOX B. RCOOH C. RCX

A. RCOX

the oxidizing agent is being A. Reduced B. Oxidized

A. Reduced the oxidizing agent steals electron from the other molecule causing its oxidation hence why it is the oxidizing agent

acid + acidic solution = A. acid ionizes less B. base ionizes more C. acid ionizes more D. base ionizes less

A. acid ionizes less there is already H3O+ in the solution so the acid does not need to dissociate much

silyl chloride, methanesulfonyl chloride, toluenesulfonyl chloride, and a diol all do what when added to an alcohol A. acts as protective groups that can be easily reverted back into an alcohol that it does not reaction with other compounds when trying to produce large molecules B. Act as strong oxidizing agents so it will oxidize a primary alcohol all the way to the carboxylic acid C. create an elimination reaction that removes the OH and creates a CC double bond

A. acts as protective groups that can be easily reverted back into an alcohol that it does not reaction with other compounds when trying to produce large molecules

the reactants NH2R, heat and carboxylic acid forms which of the following products A. amide B. ammonia C. primary amine

A. amide

Order these from least to most reactive: A. amide B. acyl chloride C. carboxylic acid D. acid anhydride

A. amide C. carboxylic acid D. acid anhydride B. acyl chloride the more reactive, the less stable (do not think about the conjugate base)or have slightly + carbons. The more reactive molecules also have better leaving groups more reactive molecules can be easily converted to less reactive molecules. less reactive molecules cannot be easily converted to more reactive molecules

the solution with the unknown concentration that is dripped into is called the A. analyate B. titrate

A. analyate this is the solution that is being analyzed

reducing sugar A. any monosaccharide w/ a hemiacetal ring B. any monosaccharide w/ a hemiketal ring

A. any monosaccharide w/ a hemiacetal ring (since they can be oxidized, they are considered reducing agents)

The process of culturing bacteria often involves inoculation of cells on a noncellular, agar-based medium. Such a methodology would NOT result in growth of animal viruses because animal viruses: A. are obligate parasites. B. lack DNA. C. assimilate carbon. D. require essential vitamin supplements for growth.

A. are obligate parasites. Animal viruses can only infect animal cells, not bacteria. Thus, animal viruses cannot grow in agar plates with bacteria.

where does oxidation occur in a galvanic cell and in an electrolytic cell? A. at the anode in both cells B. at the cathode in both cells C. at the anode in the galvanic cell and at the cathode in the electrolytic cell D. at the cathode in the galvanic cell and at the anode in the electrolytic cell

A. at the anode in both cells

what occurs if O undergoes electron capture A. becomes N B. becomes F

A. becomes N electron capture: nucleus absorbs e- which merges with a proton to become a neutron

If a leak develops in the vacuum distillation apparatus, the boiling points of the two components of caraway seed oil will: A. both increase B. both decrease C. both remain the same D. become more similar

A. both increase The boiling point of a liquid is the temperature at which the vapor pressure of the liquid equals the surface pressure. The normal boiling point is measured at 1 atm pressure. The vapor pressure of a liquid increases with increasing temperature. Hence, the boiling point of a liquid decreases as the pressure on the surface of the liquid is decreased. If a leak develops in the apparatus, the surface pressure will increase, as will the boiling points of both liquids. Thus, answer choice A is the best answer.

when 1) PBr3, Br2 2) H2O reacts with a carboxylic acid what product is formed A. bromoacetic acid B. acid bromide C. bromo acyl bromide

A. bromoacetic acid

How does IR spectroscopy work to identify the functional groups in a molecule A. certain wavelengths of radiation are absorbed by the functional groups, the light not transmitted is transmitted through the sample to a sensor that correlates the functional group B. radiation is absorbed by the groups and the wavelength not absorbed give off a certain color telling what the group is C. radiation is absorbed by the groups and the frequency of the wavelength absorbed is measured and correlated with a functional group

A. certain wavelengths of radiation are absorbed by the functional groups, the light not transmitted is transmitted through the sample to a sensor that correlates the functional group certain bonds are vibrated and bend, absorbing the radiation

Match alkoxide with its definition A. conjugate base of an alcohol B. molecule after oxidation C. molecule after reduction

A. conjugate base of an alcohol

supersaturated solution A. contains more dissolved solute than a saturated solution at the same temperature B. solute begins to precipitate out of a solution C. Just enough solute is added to dissolve in the solvent

A. contains more dissolved solute than a saturated solution at the same temperature BONUS: this occurs by heating up the solution until all the solute dissolves and then slowly recooling

epimers are a type of A. diastereomers B. anomers C. enantiomers D. stereoisomer

A. diastereomers diastereomers are stereoisomers that are not mirror images

the opposite of an enantiomer A. diastereomers B. anomers C. enantiomers D. epimers E. stereoisomer

A. diastereomers diastereomers are stereoisomers that are not mirror images

A carboxylic acid will be ____________ on an NMR spectrum A. down field B. up field

A. down field NMR detects how shielded C and H atoms are in molecules up field is more shielded down field is less shielded in carboxylic acid, EN oxygen atoms pull e- from the C and H therefore making those molecules less shielded making it down field

what isomers are shown in the pic A. functional isomers B. enantiomers C. stereoisomers D. tautomers

A. functional isomers constitutional isomers that have the same molecular formula but different functional groups so they likely have vastly different properties

The more volatile a chemical agent​ is, the more likely it is to become a: A. gas B. base C. acid

A. gas

Color arises because nickel(II) ions in a solution A. have unfilled d orbitals B. have a +2 charge C. are transition metals

A. have unfilled d orbitals The color arises because nickel(II) ion has partially filled d orbitals and the electrons in the lower energy d orbitals absorb visible light to move to the higher energy d orbitals.

Which of the following experimental modifications will most likely improve the degree of separation of two compounds in a vacuum fractional distillation A. heating the distillation flask at a slower rate B. using a vacuum source that can achieve a lower pressure inside the apparatus C. cooling the condenser with ice water D. using a shorter fractionation column

A. heating the distillation flask at a slower rate The separation of the two liquids takes place in the fractionating column as the two liquids vaporize and condense, with the lower-boiling liquid distilling first. If the fractionating column is shortened (Answer D), the liquids will vaporize and condense fewer times and the degree of separation will worsen. Cooling the condenser with ice water (Answer C) will have no effect on the degree of separation because condensation takes place after the separation. Creating a lower pressure inside the distilling apparatus (Answer B) will lower the boiling points of both liquids. Heating the distillation flask (i.e., increasing the temperature) at a slower rate (Answer A) will allow both liquids more time in the fractionating column (increase the number of theoretical plates, allowing liquid and vapor to equilibrate); therefore, of the four options, this one is most likely to improve the degree of separation of the two compounds. Thus, answer choice A is the best answer.

Which of the following properties is associated with the existence of glycine as a dipolar ion in aqueous solution? A. high dipole moment B. high molecular weight C. low dielectric constant D. low solubility in water

A. high dipole moment The problem states that glycine exists "as a dipolar ion in aqueous solution" and asks which of four properties is associated with this fact. Polarity in neutral molecules results from an uneven distribution of electron density, which can arise from separation of unlike charges. This occurs in zwitterions and in ylides. In addition, molecules that contain strongly electron-withdrawing or electron-donating substituents are highly polar and possess correspondingly high dipole moments. Thus, answer choice A is the best answer.

a high Ksp means A. high solubility B. reactant favored C. low concentration of solvent

A. high solubility Ksp is the solubility product constant

how does solute affect boiling point? A. increase B. decrease C. no change

A. increase the addition of solute to a solvent is going to increase boiling point meaning you need a higher temperature to boil solution

the first step in Mass spectroscopy is to A. ionize the molecule B. determine the number of H groups C. determine the number of Cs

A. ionize the molecule this allows us to alter the speed and direction of the molecule in the machine

what happens if O undergoes alpha decay A. its becomes C B. it becomes Ne C. it becomes O-14 D. it becomes O-18

A. its becomes C alpha decay ejects 2n and 2 p making O become C

Which measurement unit CANNOT be used to express power? A. kg•m^2•s^2 B. J•s^-1 C. ft•lb •s^-1 D. W

A. kg•m^2•s^2 A. This answer is correct because the measurement unit of power is watt, defined as J/s = ​​ft•lb/s = kg•m^2/s^3 B. This unit is consistent with the definition of the watt, which is used to express power, because by definition W= J/s ​​ C. This unit is consistent with the definition of the watt, which is used to express power, because by definition ​​W= J/s ​​= ​​ft•lb/s D. This unit is the watt, which is by definition is used to express power.

Al(OH)3(aq) + NaOH(aq) → Na[Al(OH)4](aq) In the reaction shown, Al(OH)3 acts as what kind of acid or base? A. lewis acid B. lewis base C. bronsted acid D. bronsted base

A. lewis acid Lewis acids gain electrons In the reaction shown, the Al in Al(OH)3 accepts an electron pair from the OH- of NaOH and is therefore acting as a Lewis acid. Not really acting as bronsted because not just a proton was donated

What is the geometry of the hexafluoroaluminate ion (AlF6 3-)? A. octahedral B. tetrahedral C. trigonal bipyramidal D. hexagonal

A. octahedral An ion consisting of a central metal ion and six ligands is expected to have an octahedral geometry. Thus, A is the best answer. aka square bipyramidal but I guess its not called that

2 (NH4)2HPO4(s) + H2O(l) 4 NH4+(aq) + HPO42-(aq) + H2PO4-(aq) + OH-(aq) In the equilibrium constant expression for this reaction [H2O] is omitted because the salt is: A. only weakly basic, and [H2O] is nearly constant B. strongly basic, and [H2O] is nearly zero C. only weakly acidic, and [H2O] is nearly constant D. strongly acidic, and [H2O] is nearly zero

A. only weakly basic, and [H2O] is nearly constant Aqueous acid-base equilibria very often feature water acting as either an acid or a base. It is also the solvent for aqueous systems. In the equilibrium constant expression for such reactions, water is usually omitted. The simplest explanation is that, because it is the solvent, the concentration of water stays essentially constant throughout the reaction. In such a case, that constant could be included in the value of Keq, the equilibrium constant for the reaction. The salt in the equation is (NH4)2HPO4, and the reaction shows that the substance is basic because it produced OH. Only response A includes both of these features. Thus, answer choice A is the best answer.

if the concentration of conjugate base is much higher than the concentration of the weak acid in a titration buffer solution then the pH of the buffer is A. pH > pKa B. pH = pKa C. pH < pKa

A. pH > pKa pH = pKa + log [A-]/[HA] log [10]/[1]= log 10= 1 pH = pKa + 1 pH > pKa

The initial filtration step in the glomerulus of the mammalian kidney occurs primarily by: A. passive flow due to a pressure difference. B. passive flow resulting from a countercurrent exchange system. C. active transport of water, followed by movement of electrolytes along a resulting concentration gradient. D. active transport of electrolytes, followed by passive flow of water along the resulting osmolarity gradient.

A. passive flow due to a pressure difference. There are three pressures that work together to regulate filtration in the glomerulus: glomerular capillary pressure, capsular hydrostatic pressure, and blood colloid osmotic pressure. The glomerular capillary pressure will force filtrate from a capillary into Bowman's capsule; the other two forces promote movement of the filtrate in the opposite direction.

What solvent favors SN2 reactions? BONUS: how is it helpful A. polar aprotic solvent B. polar protic solvents

A. polar aprotic solvent this help by not getting involved in the reaction (DMSO, acetone)

which has the highest BP A. primary amine B. secondary amine C. tertiary amine

A. primary amine primary amine has 2 H and 1 R group making it have the highest H bonding and the highest BP

A spontaneous reaction favors A. products B. reactants

A. products they have lower energy

if Keq is more than 1 A. products are favored B. reactants are favored

A. products are favored top number is much larger

Rank the following molecules in order of decreasing oxidation. A. propanoic acid B. Propane C. Propanol D. propanal

A. propanoic acid D. propanal C. Propanol B. Propane oxidation is higher with the more carbon oxygen bonds

Why are boiling chips used during distillation? A. provide nucleation sites that give the liquid a place to start forming bubbles to prevent superheating B. lower the boiling points of the substances to be distilled C. lower the pressure of the substances to be distilled which lowers the boiling point

A. provide nucleation sites that give the liquid a place to start forming bubbles to prevent superheating create surface area for small air bubbles to form, to prevent violent bumping of distillate, to speed up the distillation

which of the following is not a stereochemical labeling system A. r/s B. R/S C. D/L D. d/l

A. r/s R/S- standard was of classifying one chiral center D/L- used for carbs, D/L glucose d/l- polarized right rotation

If Q is less than Keq A. reaction proceeds forwards producing more product B. reaction proceeds backwards producing more reactants

A. reaction proceeds forwards producing more product Ex: Q = 1/2, Keq = 1/1 Q is less than Keq to get to Keq from Q the 1 at the top of Q must become 2, this required as increase in products

Buffers A. resistant pH change B. Cause pH change C. help pH change

A. resistant pH change

Atoms located upfield on an NMR spectroscopy chart are more A. shielded by electron density B. deshielded

A. shielded by electron density come to resonance at a lower frequency how often the atom switches between a high energy state and a low energy state in a magnetic field (this is called resonance)

saponification is used to make what? select all that apply A. soaps B. alcohol C. carboxylate salt

A. soaps B. alcohol C. carboxylate salt carboxylate salt is soap

What isomers are shown in the pic A. stereoisomers B. constitutional isomers

A. stereoisomers Compounds with the same structural formula but with a different arrangement of the atoms in space.

What type of isomer is pictured: select all that apply A. stereoisomers B. Configurational isomers C. conformational isomers

A. stereoisomers C. conformational isomers Conformational isomers are a type of stereoisomers that differ by rotation about one or more single bonds, usually represented using Newman projections.

a more positive reduction potential means(select all) A. that chemical is more likely to be reduced B. that chemical is less likely to be reduced C. more spontaneous reductions D. less spontaneous reductions

A. that chemical is more likely to be reduced C. more spontaneous reductions

what peak on Mass spectroscopy is most important A. the peak furthest right B. the peak furthest left C. the largest D. the smallest

A. the peak furthest right this is the peak of the molecular ion/parent ion this peak tells us the molecular weight of the molecule which is want we wanted to know Ex: 170 in pic

The N in the molecule N-methylpropanamide is there to show A. there is a bonded atom on the amide group B. there is a methyl group on the N terminus of an amino acid C. there is a N bonded to a carbon

A. there is a bonded atom on the amide group occurs for secondary and tertiary amides

which direction does a dextrorotatory system rotate light A. to the right (clockwise) B. to the left (counterclockwise) C. neither

A. to the right (clockwise) AKA the d/l system this is a compound that rotates the plane of polarized light to the right

What amino acid absorb UV light the best during a UV spectroscopy A. try B. asp C. his

A. try conjugated pi system have closer electron orbitals, the allows electrons to be more easily excited by the UV radiation

when the chemical ubiquinone(coenzyme Q) is reduced it becomes A. ubisemiquinone B. ubiquinol

A. ubisemiquinone ubiquinol is when both ketones are reduced

ordinate of a graph A. y axis B. x axis C. y intercept D. plotted line

A. y axis

What molecule is this

ATP

Acid Catalyzed keto-enol tautomerization vs base catalyzed keto-enol tautomerization

Acid Catalyzed: protonation occurs first base catalyzed: protonation occurs last

Base catalyzed Aldol Condensation Reaction

Aldehyde + Ketone --> ketone+aldehyde + ketone+ketone side product alpha H removed double bond between alpha C and carboxyl C, double bond moved up to O as lone pair forming enolate double bond between alpha C and carboxyl C attacks double bond of ketone or aldehyde enolate is protonated H removed from alpha C forming double bond between alpha and beta carbon kicking off the OH group

Are aldehydes or ketones more reactive?

Aldehydes because less steric hindrance and fewer electron-donating alkyl groups, they are more acidic

Match each type of carbohydrate with its feature Aldose, ketose, pentose, furanose, pyranose A. Sugar in a 5 membered ring B. A carbohydrate whose carbonyl group is an aldehyde C. 5 carbons in the chain D. carbonyl group in middle of carbon chain D. Sugar in a 6 membered ring

Aldose: B. A carbohydrate whose carbonyl group is an aldehyde Ketose: D. carbonyl group in middle of carbon chain hint: this is a ketone Pentose: C. 5 carbons in the chain Furanose: A. Sugar in a 5 membered ring Pyranose: D. Sugar in a 6 membered ring

ammonium vs ammonia

Ammonia is un-ionized, and has the formula NH3. Ammonium is ionized, and has the formula NH4+.

gas-liquid chromatography

An advanced form of chromatography that uses an inert gas as the mobile phase and a column of gel-coated beads as the stationary phase.

how are predictions of a ideal gas different from predictions of a real gas how do these differ at STP from extreme conditons

At STP, real gases have slightly smaller but almost equivalent pressure At high temps, real gases have much higher pressure at low temps, real gases have a much smaller pressure

Suppose that a blood vessel of cross-sectional area A carries microbubbles at a speed v into a capillary bed. If the capillary bed is made up of n capillaries, each with cross-sectional area a, with what speed will the blood flow in the capillary bed? A. (na/A)v B. (A/na)v C. (a/A)v D. (nA/a)v

B. (A/na)v The answer to this question is B because, according to the continuity equation (A1v1 = A2v2), if u is the speed of flow in the capillary bed, then A × v = n × a × u, so u = A × v/(n × a).

What is the molar concentration of Na+(aq) in a solution that is prepared by mixing 10 mL of a 0.010 M NaHCO3(aq) solution with 10 mL of a 0.010 M Na2CO3(aq) solution? A. 0.01mol/L B. 0.015mol/L C. 0.02mol/L D. 0.03mol/L

B. 0.015mol/L Since 1 equivalent of NaHCO3 provides 1 equivalent of Na+, the molar concentration of Na+(aq) in 0.010 M NaHCO3(aq) solution is also 0.010 M = 0.010 mol/L. The molar concentration of Na+(aq) in 0.010 M Na2CO3(aq) solution is 0.020 mol/L since 1 equivalent of Na2CO3 provides 2 equivalents of Na+. When equal volumes of these two solutions are mixed, the resulting molar concentration is equal to their average, this is because 0.010 mol/L + 0.020 mol/L = 0.030 mol/2L = 0.015mol/L Thus, B is the correct answer.

When glycerol reacts with three different fatty acids, how many stereogenic centers does the product triacylglycerol contain? A. 0 B. 1 C. 2 D. 3

B. 1 only the middle C center of glycerol is chiral

guanine contains (select all) A. 1 H donor B. 1 H acceptor C. 2 H donors D. 2 H acceptors

B. 1 H acceptor C. 2 H donors

which if these reactants turns carboxylic acid into a primary alcohol A. PCC B. 1)LAH 2)H3O+ C. KMnO4

B. 1)LAH 2)H3O+ first LAH turns carboxylic acid into an aldehyde then attacks against to get a -O. Then an H is stollen from H3O+ by the -O to make an OH

an object O is at a distance of three focal lengths from the center of a convex lens. What is the ratio of the height of the image to the height of the object? A. 1/3 B. 1/2 C. 2/3 D. 3/2

B. 1/2 M = hi/ho = -di/do The ratio of the image height to the object height is equal to the ratio of the lens-image distance to the object-lens distance. According to the thin lens equation, the distance between the lens and the image is ​​1/f = 1/3f + 1/di solve for di and get (3/2)f. then used the magnification equation ((3/2)/3)) for di/do = 1/2 = hi/ho

A 50-kg child is riding on a playground merry-go-round. If the radius of the circular path of the merry-go-round is 5.0 m and the frequency is 0.1 hertz, what is the force required to keep the child on the ride? A. 25N B. 100N C. 1000N D. 2500N

B. 100N If the frequency of the merry-go-round is 0.1 Hz, then the period is 10 s. We can calculate the speed of the merry-go-round from V = 2πr/T = 2(3.14)(5.0 m)/(10 s) = 3.14 m/s. Using this in the centripetal force equation: Fc = mv^2/r = (50 kg)(3.14 m/s)^2/(5.0 m) Approximate the square of the speed at 10 m^2/s^2. Fc = (50)(10)/(5) = 100 N

A single sports fan is capable of yelling at an intensity level of 80 dB from a given distance. If 10,000 similar fans were all yelling from the same distance, which of the following would be closest to the observed intensity level? A. 84 dB B. 120 dB C. 160 dB D. 320 dB

B. 120 dB 10,000 fans would be capable of yelling at an intensity 10,000, or 10^4, times greater than the single fan alone. According to the decibel scale, this would correspond to an increase of 40 dB. also dB = 10log(I/Io) = 10log(10000/1) = 10log(10000) 10^(dB/10) = 10,000 exponent needs to be 4, what dB would give an exponent of 4, 40 so 80 + 40 = 120

functional groups with C double bonds normally peak around which wavelength on an IR spectroscopy A. 1000-1200 B. 1600-1800 C. 2100-2300 D. 3000-3500

B. 1600-1800 these normally have a small wavelength less interaction so more transmittance through the sample

When two amino acids are joined via a peptide bond, what is the mass of the byproduct of this reaction? (Note: Assume that the amino acids were not modified by protecting groups.) A. 17 amu B. 18 amu C. 32 amu D. 44 amu

B. 18 amu

A woman is standing two meters away from a speaker. The speaker is turned up to 11 on its dial, and the power of the sound hitting her tympanic membrane increases 100-fold. How far would the woman need to move to reduce the sound back to its original decibel level? A. 10 m B. 18 m C. 20 m D. 50 m

B. 18 m The decibel level of sound is given by dB = 10log (I/Io), where I denotes intensity and Io is the reference intensity, which represents the lowest intensity audible to the human ear. In order to have the same decibel level, the same intensity, I, must reach the tympanic membrane. The intensity of sound is given by the equation I = P/A, where P = power and A = area. If the power increases 100-fold, then the area would need to increase by a factor of 100 to maintain the same intensity. The area, A, of the sound wave is given by the surface area of a sphere as the sound travels away from the speaker. The surface area of a sphere is proportional to the radius squared, or r^2 (A = 4πr^2). So, in order to increase the area 100-fold, the radius, r, must increase by a factor of 10 (10^2 = 100). Thus, if the woman originally stands two meters away then to maintain the same decibel level, she needs to stand 2 x 10 = 20 m away from the speaker. Standing 2 m away, the woman must move 18 m to reach a total final distance of 20 m (choice C is a clever trap).

how many equivalents point will diprotic acids have A. 1 B. 2 C. 3

B. 2 Since they are diprotic, they can lose 2 protons and therefore have two equivalents point

which of these if injected into a gas-liquid chromatography, would create the first peak observed? A. 2-Methyl-2-butanol B. 2-Methyl-2-butene C. 2-Chloro-2-methylbutane D. 2-Bromo-2-methylbutane

B. 2-Methyl-2-butene In gas-liquid chromatography, the first peak to emerge will be from the least polar, most volatile compound. 2-Methyl-2-butene is nonpolar and the most volatile because it only has London forces for intermolecular attraction.

how many NMR H1 signals would be in this compound 3H-C-CH2-O-CH3 A. 1 B. 3 C. 6 D. 8

B. 3 Hs are grouped by the C atom they are attached to, not the number of H atoms for NMR C12, the number of carbon atoms is counted as groups TMS in picture is not part of the results

N-H on IR spectrum A. 1650 to 1750 B. 3200 to 3500 sharp peak C. 3200 to 3500 broad peak D. 2500 to 3300 and 1750

B. 3200 to 3500 sharp peak

cross aldol condensation can produce how many products A. 2 B. 4 C. 6 D. 8

B. 4

When choosing a buffer to use for an experiment conducted at pH 5.3, it would be best to choose one with a pKa of: A. 2.14 B. 4.75 C. 6.5 D. 7.0

B. 4.75 Good experimental design protocols state that a good buffer has a pKa within 1 pH unit of the desired experimental conditions. Within this range, there is less than a 10 to 1 ratio of acid to base or base to acid forms of the buffer, and so some buffering capacity still remains.

Under what set of conditions would an air embolism be MOST likely to form in blood? A. 42°C and PO2 = 300 mmHg B. 42°C and PO2 = 120 mmHg C. 13°C and PO2 = 120 mmHg D. 13°C and PO2 = 300 mmHg

B. 42°C and PO2 = 120 mmHg an air embolism is caused when a bubble of air is formed in the blood and blocks a vessel. If a gas is insoluble in a fluid, it can escape as a bubble (think of warm soda going flat). Gases generally have greater solubility in cooler liquids. Also, the solubility of a gas increases as the partial pressure of the gas increases. This is primarily due to the hydrogen bonding effects of aqueous fluids that produce cavities within the fluid; these cavities can accommodate the gas molecules. Thus, we would expect an air embolism to form when gas is insoluble, which occurs in high-temperature, low-pressure systems.

HCO3-(aq) + H3O+(aq) H2CO3(aq) + H2O(l) What is the pH of a buffer solution that is 0.2 M in HCO3- and 2 M in H2CO3? (Note: The first pKa of carbonic acid is 6.37.) A. 4.37 B. 5.37 C. 6.37 D. 7.37

B. 5.37 The pH of the solution can be calculated using the Henderson-Hasselbach equation: pH = pKa + log([base]/[acid]). Plugging in the values provided in the question gives pH = 6.37 + log(0.2/2) pH - 6.37 = log(0.2/2) pH - 6.37 = log(0.1) 10^ (pH - 6.37) = 0.1 0.1 is 1/10 so we want the exponent to be -1 therefore pH must be 5.37

A certain bacterium was cultured for several generations in medium containing 15N, transferred to medium containing 14N, and allowed to complete two rounds of cell division. Given that the bacterium's genome mass is 5.4 fg when grown in 14N media and 5.5 fg when grown in 15N medium, individual bacteria with which of the following genome masses would most likely be isolated from this culture? A. 5.4 fg only B. 5.4 fg and 5.45 fg C. 5.4 fg and 5.5 fg D. 5.45 fg only

B. 5.4 fg and 5.45 fg DNA replication occurs in a semiconservative way and, after two rounds of bacterial growth, there will be bacteria containing only 14N and bacteria containing the same amount of 15N and 14N. Thus, the genome mass will be 5.4 fg for the bacteria that only contain 14N, and 5.45 fg for the bacteria that contain equal levels of 14N and 15N.

If 10 moles of H3PO4 are added to 5 liters of water, what is the normality of the solution? A. 2 N B. 6 N C. 30 N

B. 6 N N = moles of H+/L when completely dissociated, H3PO4 makes 3 H+ ions 10 moles per ion therefore 30 moles of H+ 30 mol/5L = 6 N or N = number of H+/OH- * M 3 H+ ions M = mol/L = 10/5 = 2 N = (3)(2) = 6 N

the equivalence point of a strong acid/strong base titration is A. < 7 B. 7 C. > 7

B. 7 the only products are H2O, salt, and the conjugate base H2O and salt are neutral the conjugate base of strong acids is so weak that it has no effect on pH

Two enzymes identified after separation by affinity chromatography include AST and ALT. Previous research indicates that AST has a larger molecular mass and a larger physical size than does ALT. An aqueous mobile phase was used to migrate these through an agarose gel using a technique known as gel filtration chromatography, which is a category of size-exclusion chromatography. What was the likely outcome? A. ALT migrated farther than AST due to the agarose pore system's greater effect on larger molecules compared to smaller ones. B. AST migrated farther than ALT due to the agarose pore system's greater effect on smaller molecules compared to larger ones. C. AST and ALT migrate with equal rates, because in size-exclusion chromatography, there is no interaction between analytes and the surface of the stationary phase. D. AST and ALT migrate with equal rates, because in gel filtration there are equal effects on molecules of equal charge.

B. AST migrated farther than ALT due to the agarose pore system's greater effect on smaller molecules compared to larger ones. in size exclusion chromatography, the larger molecules don't interact with the pores as much therefore exit the tube first

What does PCC do? A. Acts as a reducing reagent to reduce aldehydes into primary alcohols B. Acts as an oxidizing reagent to oxidize primary alcohols to aldehydes and secondary alcohols to ketones C. Turns triple bonded carbons into double bonded carbons

B. Acts as an oxidizing reagent to oxidize primary alcohols to aldehydes and secondary alcohols to ketones

If no sulfate, nitrate, or other electron acceptors are present, fermentation will be the main process Which of the following species, if present, will most likely result in fermentation? A. AlCl3 B. CN- C. NO3- D. PO43-

B. CN- For fermentation to predominate, we need a lack of available electron acceptors. Lewis bases are species that donate an electron pair. Thus, they are most likely to cause fermentation. Only choice B is a Lewis base.

crystalline form has two formula units per unit cell. Since the formula unit for hydroxyapatite is Ca5(PO4)3OH, What is the formula of hydroxyapatite that is present in the unit cell of the crystalline material? A. Ca5(PO4)3OH B. Ca10(PO4)6(OH)2 C. Ca3(PO4)2 D. Ca(OH)2

B. Ca10(PO4)6(OH)2

Nicotinamide nucleotides are neither oxidized nor reduced during which step of cellular respiration? A. Glycolysis B. Chemiosmosis C. Citric acid cycle D. Electron transport chain

B. Chemiosmosis Chemiosmosis is ATP synthesis via ATP synthase Chemiosmosis is the only step of cellular respiration where NAD+ is neither reduced to form NADH, nor is NADH oxidized to form NAD+.

what is the empirical formula of carbohydrates A. Cm(H30)n B. Cm(H20)n C. CmH20n D. (CH2O)m

B. Cm(H20)n think C6H12O6

Chiral molecules are what type of isomer A. stereoisomers B. Configurational isomers C. conformational isomers D. functional isomers E. enantiomers

B. Configurational isomers it is an actual isomer because you cant just flip the molecule over, it had to be mirrored. To do this bonds must be broken (Configurational isomers)

What type of isomer is pictured: select all that apply A. stereoisomers B. Configurational isomers C. conformational isomers

B. Configurational isomers Can only be interchanged by breaking and reforming bonds if you look at the bonded atoms, it is clear that the D and H had to be broken and switched to create the isomer

Antioxidant activity is an important function of which of the following vitamins? A. B1 B. E C. D D. K

B. E Vitamin E refers to a set of closely-related, lipid-soluble compounds that function as antioxidants. Additional possible functions of vitamin E are under investigation, but for the purposes of the MCAT it is primarily considered an antioxidant.

Absorption of ultraviolet light by organic molecules always results in what process? A. Bond breaking B. Excitation of bound electrons C. Vibration of atoms in polar bonds D. Ejection of bound electrons

B. Excitation of bound electrons The absorption of ultraviolet light by organic molecules always results in electronic excitation. Bond breaking can subsequently result, as can ionization or bond vibration, but none of these processes are guaranteed to result from the absorption of ultraviolet light. It is a Knowledge of Scientific Concepts and Principles question because you must recognize a correct scientific principle.

When is ∆H negative? A. Endothermic B. Exothermic

B. Exothermic heat released by the system

When a solute is added to a solvent, the boiling point of the solution is A. Lower B. Higher C. the same

B. Higher A solute has a vapor pressure of zero, so the vapor pressure of the solution is less than the vapor pressure of the solvent. Thus, a higher temperature is needed for the vapor pressure to reach the surrounding pressure, and the boiling point is elevated

The results of a separate study of the thermodynamic parameters for the interactions of proteins with cyclohexanol and quaternary ammonium salts indicate that the hydrophobic solute-solute interaction is spontaneous, and that ΔH and ΔG have opposite signs. Which of the following must be true for this interaction when temperature is a positive value? I. ΔH > ΔG II. 0 < ΔS III. ΔH < ΔS A. I only B. I and II only C. II and III only D. I, II, and III

B. I and II only For the interaction to be spontaneous, the free energy change of the reaction must be negative. The question states that the enthalpy and free energy changes for the interaction have opposite signs. Thus, ∆G is negative and ∆H is positive (I). For a reaction to be both endothermic (positive ∆H) and spontaneous (negative ∆G), the reaction must involve an increase in entropy (∆S is positive) (II). This is according to the equation: ∆G = ∆H - T∆S.

The Arrhenius equation, k = Ae^(-Ea/(RT)), expresses the relationship between the rate constant, k, and the temperature of a reaction. According to this equation, which of the following will increase the rate of a reaction? A. Decreasing the pre-exponential factor, A B. Increasing the temperature, T C. Increasing the activation energy, Ea D. Decreasing the temperature, T

B. Increasing the temperature, T While this question introduces the Arrhenius equation, you can answer it simply by remembering the principle in general chemistry that increasing temperature increases the rate of reaction. However, notice that increasing T causes the exponent of e to be less negative, so k increases.

In order to measure the ∆G, researchers needed to denature the proteins. Which of the following steps would be LEAST suitable for this procedure? A. Increasing temperature B. Irradiating the protein with non-ionizing radiation C. Adding a concentrated chaotropic agent D. Lowering the pH

B. Irradiating the protein with non-ionizing radiation This answer choice specifically says that the protein will be irradiated with nonionizing radiation, which does not break bonds. Therefore, this method would not be able to denature the proteins. Irradiating the proteins with X-rays or gamma rays certainly can denature them, but these are forms of ionizing radiation.

[Cu(H2O)4]2+(aq) + 4NH3(aq) <==> [Cu(NH3)4]2+(aq) + 4H2O(l) Consider the reaction shown in the equation at equilibrium. Would the concentration of [Cu(NH3)4]2+ increase if the equilibrium were disturbed by adding hydrochloric acid? A. Yes, because the equilibrium in Equation 1 would shift to the left B. No, because the equilibrium in Equation 1 would shift to the left C. Yes, because the equilibrium in Equation 1 would shift to the right D. No, because the equilibrium in Equation 1 would shift to the right

B. No, because the equilibrium in Equation 1 would shift to the left Hydrochloric acid will protonate ammonia in a Brönsted acid-base reaction and reduce the amount of ammonia present. The disturbed equilibrium responds in a way to restore ammonia, but this causes the amount of [Cu(H2O)2(NH3)2]2+ to decrease.

when a primary alcohol reacts with a ketone what produce is produced A. RC(OR)2H B. RC(OR)2R C. RCOH

B. RC(OR)2R ketal

Under anaerobic conditions, bacteria can sometimes derive energy from the oxidation of sulfur-containing species. Sulfur in which of the following compounds would be LEAST likely to be oxidized by an anaerobic bacteria? A. H2S (aq) B. SO4^2- (aq) C. S2O3^2- (aq) D. S8 (s)

B. SO4^2- (aq) Of the answer choices, the oxidation number for sulfur would need to be in a reduced form. The highest oxidation state for sulfur is +6, corresponding to the loss of all of its valence electrons. The oxidation number for sulfur (x) in sulfate, SO4^2-, can be determined by assuming that the oxygen atoms are oxides having a -2 charge and that the sum of the oxidation numbers must equal the overall charge of the ion, giving -2 = x + 4(-2) -2 = x - 8 +6 = x The oxidation number of sulfur in sulfate is +6, and choice B is the best answer. The oxidation number of sulfur in hydrogen sulfide (H2S), thiosulfate (S2O3^2-) and yellow elemental sulfur (S8) are -2, +2 and 0, respectively, all of which could be oxidized and be a source of energy for anaerobic bacteria.

Which of the following best explains why arginine is more basic than lysine? A. The electron-donating groups around the basic nitrogen on arginine make its conjugate base less stable. B. The electron-donating groups around the basic nitrogen on arginine make its conjugate acid more stable. C. The lack of electron-donating groups on lysine make its conjugate acid more stable. D. The lack of electron-withdrawing groups on lysine make its conjugate base more stable.

B. The electron-donating groups around the basic nitrogen on arginine make its conjugate acid more stable. This question is asking us to determine why arginine is more basic than lysine. The reason must be related to how arginine is better able to handle being protonated, as this is the essence of being a base. Since, in its protonated form, arginine has electron-donating groups via resonance with other nitrogens, it is a more stable conjugate acid. The resonance structures of arginine at this position are shown below. Note that the backbone amino and carboxylic acid groups are deprotonated, meaning that this is the structure of arginine at relatively high pH (albeit not high enough to deprotonate the side chain).

How will the rate of a catalyzed reaction be affected if the solid catalyst is finely ground before it is added to the reaction mixture? A. The rate will be faster because a greater mass of catalyst will be present. B. The rate will be faster because a greater surface area of catalyst will be exposed. C. The rate will be slower because the fine catalyst particles will interfere with product formation. D. The rate will remain the same because the mass of catalyst will be the same.

B. The rate will be faster because a greater surface area of catalyst will be exposed.

A circuit is comprised of a 12-volt battery with three light bulbs with the same resistance. If two of the light bulbs are wired in parallel and are in series with the third light bulb, what happens when one of the parallel light bulbs is unscrewed? A. All of the light bulbs remain lit. B. Two of the light bulbs remain lit. C. One of the light bulbs remains lit. D. All of the light bulbs go out.

B. Two of the light bulbs remain lit. Even after unscrewing one of the parallel light bulbs, there is still a circuit for current to flow from the positive terminal to the negative terminal of the battery through the two remaining light bulbs.

If an IR absorption spectrum were obtained for the dimer in Figure 1, a strong signal at 1750 cm^-1 would most likely indicate the presence of. You don't need to see the picture to know the spectrum values A. an O-H bonds. B. a C=O bonds. C. a C≡C bonds. D. a C-O bonds.

B. a C=O bonds. C=O bond stretching typically exhibits a frequency range of 1650-1750 cm-1. O-H bond stretching in a carboxylic acid typically exhibits a frequency range of 2500-3300 C≡C bond stretching typically exhibits a frequency range of 2100-2260 C-O bond stretching typically exhibits a frequency range of 1000-1320 cm

which of the following is the name of a compound that can both donate and accept protons A. mixed species B. amphoteric C. amphiphilic

B. amphoteric

where does reduction occur in a galvanic cell and in an electrolytic cell? A. at the anode in both cells B. at the cathode in both cells C. at the anode in the galvanic cell and at the cathode in the electrolytic cell D. at the cathode in the galvanic cell and at the anode in the electrolytic cell

B. at the cathode in both cells

base + acidic solution = A. acid ionizes less B. base ionizes more C. acid ionizes more D. base ionizes less

B. base ionizes more there is more acid in the solution (acidic) and the base will dissociate more to cancel it out

the ionization constant of the weak acid, H2S is: A. < 0 B. between 0 and 1 but closer to 0 C. between 0 and 1 but closer to 1 D. > 1

B. between 0 and 1 but closer to 0 H2S was described as being a weak acid. This means that a small fraction of the solute molecule ionize and release protons into solution. The value of the ionization constant is therefore somewhere between 0 and 1, but much closer to 0. As an example, phosphoric acid, among the strongest of the weak acids, has an ionization constant that is approximately 1 x 10^-2, or 0.01. This is much closer to 0 than 1.

What isomers are shown in the pic A. stereoisomers B. constitutional isomers

B. constitutional isomers same molecular formula, different connectivity this includes tautomers and functional isomers

lactone A. cyclic ketone B. cyclic ester C. cyclic carboxylic acid

B. cyclic ester named according to the number carbon atoms other than the carbonyl carbon that the O is attached to 3- alpha, 4-beta, 5-gamma, etc

if solute is added to one side of solvent with a semipermeable membrane between what happens to osmotic pressure? A. increase B. decrease C. no change

B. decrease if solute is added to one side, osmosis will cause water to flow towards to side with solute. osmotic pressure needs to decrease on that side to stop the flow of water after it has reached equilibrium on both sides

when you add solute to a solution, what happens to vapor pressure? A. increase B. decrease C. no change BONUS: why?

B. decrease this is because solute will have interactions with the solvent making it more difficult to evaporate. This also occurs because the top of the solution has solute particles too so there is less room for solvent to evaporate out

how does solute affect freezing point? A. increase B. decrease C. no change

B. decrease Hint: this is why we put salt on the roads freezing, or becoming solid, creates order and decreases entropy(the universe does not like this). Solutions have high entropy because of the mix of solvent and solute, so it takes more energy to decrease their entropy to the same point.

Gamma rays are A. electrons B. electromagnetic waves C. positrons

B. electromagnetic waves

In a buffer solution where the weak acid([HA]) and its conjugate base([A-]) have the same concentration or moles the pH of the solution is A. 0 B. equal to pKa C. equal to the exponent(x) of the concentration (1 * 10^x)

B. equal to pKa pH = pKa + log [A-]/[HA] the log of 1 = 0 so pH = pKa + 0

IR spectroscopy measures A. light polarization of a molecule B. functional groups of a molecule, this can also help identify the molecule C. molecular weight of the molecule D. boiling point of the molecule, this can also help identify the molecule depending

B. functional groups of a molecule, this can also help identify the molecule

What is the difference between furanose and pyranose carbohydrates A. Furanose is formed from a 5 C chain and pyranose is formed from a 6 carbon chain B. furanose is a 5 membered ring and pyranose is a 6 membered ring both formed from a 6 C chain C. furanose is formed when the OH on the amomeric C points down and pyranose is formed when the OH on the anomeric carbon points up

B. furanose is a 5 membered ring and pyranose is a 6 membered ring both formed from a 6 C chain

What environments create more ideal gases A. high pressure B. high temperatures C. High volume D. low pressure

B. high temperatures C. High volume D. low pressure large hot open space with low pressure these are the conditions that favor the formation of a gas in the first place

molecular ion/parent ion A. ion generated that has lost all electrons B. ion generated that has lost only one electron C. ion generated after it has been protonated

B. ion generated that has lost only one electron H3C-CH3 becomes [H3C-CH2]+

what happens if O undergoes beta plus decay A. its becomes N+ B. it becomes N- C. it becomes F+ D. it becomes F-

B. it becomes N- beta plus decay converts a p into a n making O become N, there are now less p so it is negative

central phase in experiments is A. hydrophilic B. lipophilic

B. lipophilic

What happens to a reaction when we increase the volume of a container A. Total pressure of the container will decrease B. partial pressure of each gas will change but the total pressure will stay the same C. Total pressure of the container will increase

B. partial pressure of each gas will change but the total pressure will stay the same this causes a reduction in pressure which would cause the system to want to increase pressure to do this the equation shifts to the side that has more moles of GAS, GAS ONLY!!! allowing the pressure to increase Hint: adding an inert gas will increase the total pressure of a system but this will not have any influence on the partial pressure of each gas so the equilibrium will not shift

What solvent favors Sn1 reactions? BONUS: how is it helpful A. polar aprotic solvent B. polar protic solvents

B. polar protic solvents these are solvents that form hydrogen bonds (water, ethanol, methanol) these help the leaving groups not rejoin the reaction they just left Ex: if Br leaves it could just attack the carbonation but the Hs hydrogen bond to the Br making it more stable

A nonspontaneous reaction favors A. products B. reactants

B. reactants they have lower energy

if Keq is less than 1 A. products are favored B. reactants are favored

B. reactants are favored the bottom number is larger

what is Ksp A. equilibrium constant B. solubility product constant C. acid dissociation constant

B. solubility product constant

what factors influence Kw A. dissolved gas B. temperature C. solution

B. temperature Kw: water autoionization, water reacts with itself to form ions (H3O+, OH-) as temp increases the reaction increases and more products are made

the strength of an acid tell us about A. the acids ability to create OH ions B. the acids ability to dissociate C. the acids ability to react with water

B. the acids ability to dissociate strong acids dissociate more and have a larger Ka value the same for bases as well

What is true of capillary action, select all that apply A. It is a cohesive force B. it is the attraction of the surface of a liquid to the surface of a solid C. it causes the edges of water in a tube to rise

B. the attraction of the surface of a liquid to the surface of a solid C. it causes the edges of water in a tube to rise Not A because that would be between molecules of the same substance

Efforts to treat lactic acid buildup in muscles were attempted using dissected muscle specimens in the laboratory. One of these experiments involved ammonium formation from dissolved ammonia. Under conditions of excessive lactic acid: A. the final concentration of ammonium will be higher than otherwise due to higher pH. B. the final concentration of ammonium will be higher than otherwise due to lower pH. C. the final concentration of ammonium will be lower than otherwise due to higher pH. D. the final concentration of ammonium will be lower than otherwise due to lower pH.

B. the final concentration of ammonium will be higher than otherwise due to lower pH. In an acidic environment, a base such as ammonia (NH3) will dissolve into its conjugate acid, ammonium (NH4+), to a greater extent than would have been the case in a neutral or a basic environment. An environment with lower pH is an environment that is more acidic.

In locations with very low dissolved O2 concentrations in drinking water, the observed Ksp of ferrous (II) hydroxide will be: A. slightly smaller. B. the same. C. slightly larger. D. much larger.

B. the same. what does O2 in solution have to do with the solubility of a compound? it has nothing to do with it Equilibrium constants, including Ksp, are not affected by concentration

Nucleation A. when alpha rays are used to tell the amount of neutrons in a nucleus of an atom B. the step that occurs prior to crystallization where small molecules are used to give the crystals a place to form C. the reaction that occurs when certain infectious bacteria disable the nucleus structure causing apoptosis.

B. the step that occurs prior to crystallization where small molecules are used to give the crystals a place to form

the solution with a known concentration that is in the buret is called the A. analyate B. titrate

B. titrate this is the one that is dripped

which direction does a levorotatory system rotate light A. to the right (clockwise) B. to the left (counterclockwise) C. neither

B. to the left (counterclockwise) AKA the d/l system this is a compound that rotates the plane of polarized light to the left

What is enthalpy (H)? A. the energy needed to raise the temp of one gram of liquid water by one degree celsius B. total heat content of a system C. A measure of disorder or randomness D. heat released by a system

B. total heat content of a system

radioactive decay A. an atom loses a proton for every electron lost B. unstable isotopes break down C. an atom loses neutrons decreasing its mass

B. unstable isotopes break down when this occurs mass, energy, or photons can be released

the photoelectric effect A. when light hits electrons they become excited and absorb the energy eventually they will go back to ground state and eject a photon B. when light hits electrons, the electrons are excited and get ejected from their atoms C. when light hits electrons in chloroplasts, the electrons are excited and ATP can be made

B. when light hits electrons, the electrons are excited and get ejected from their atoms

abscissa of a graph A. y axis B. x axis C. y intercept D. plotted line

B. x axis

match each gas law with its definition Boyles, Charles, avogadros A. volume of gas is directly related to the number of moles of gas B. Volume and temp of a gas are direct related C. pressure and volume are inversely related D. the pressure of a gas is direct proportional the moles of a gas

Boyles: C. pressure and volume are inversely related P1V1=P2V2 Charles: B. Volume and temp of a gas are direct related V1/T1=V2/T2 Avogadros: A. volume of gas is directly related to the number of moles of gas V1/n1=V2/n2

What is the oxidation number of aluminum in Na[Al(OH)4](aq)? A. +1 B. +2 C. +3 D. +4

C. +3 The oxidation number of Na is +1, and the charge on a hydroxide ion is -1. There are 4 OH- so = -4 For the compound to be neutral, aluminum would have to have an oxidation number of +3. Thus, C is the best answer.

pH of 10M HCl A. 0.5 B. 1 C. -1

C. -1 HCl is a strong acid which would dissociate into H+ and Cl- this creates a 10M of H+ pH = -log[H+] pH = -log(10) 10^x(this is pH) = -10 x = -1

A horizontal force of 100 N is applied to a 50-kg box that is accelerating at 1 m/s2 on a rough, horizontal surface. What is the work done by kinetic friction if the box is moved 4 m? A. 0 J B. -150 J C. -200 J D. -400 J

C. -200 J Fnet = ma = Fapplied - Fk, where Fk is the force of kinetic friction. Using this formula: (50 kg)(1 m/s2) = 100 N - Fk 50 N = 100 N - Fk Fk = 50 N To calculate work, we use W = Fd cos θ. W = (50 N)(4 m)(cos 180º) = -200 Nm = -200 J.

in the amino acid KTFCGPEYLA what is the net charge at a pH of 7.2 A. -2 B. -1 C. 0 D. +1

C. 0 K-lysine E- glutamic acid

The enzyme glutathione reductase catalyzes the conversion of the molecule GSSG to GSH, which is essential in order for cells to counteract the effects of reactive oxygen species in the cytoplasm. Glutathione reductase cleaves a single disulfide bond within the GSSG molecule, resulting in two GSH molecules. Solely using this given information, how does the oxidation number change from a single GSSG molecule to a single GSH molecule? A. +2 to 0 B. +1 to 0 C. 0 to -1 D. 0 to -2

C. 0 to -1 In a disulfide S-S bond, the two sulfur atoms share their covalent electrons equally and therefore have an oxidation number of zero. When the disulfide bond gets reduced to two -SH sulfhydryl groups, S-H covalent electrons spend their time more around the sulfur in the GSH molecule, as opposed to the hydrogen. This is because sulfur is more electronegative than hydrogen. As a result, the sulfur in GSH has an oxidation state of -1. All else being equal, this suggests that a GSSG molecule has an oxidation state of 0, while a GSH molecule has an oxidation state of -1.

What is the normality of a 0.015 M solution of phosphoric acid? A. 0.015 N B. 0.030 N C. 0.045 N D. 0.060 N

C. 0.045 N The chemical formula of phosphoric acid is H3PO4. Normality, in the context of acids, refers to the number of moles of protons per liter of solution (in other words, to the "molarity of protons"). Normality can be calculated by multiplying the molarity of the solution by the number of protons per molecule of acid (here, 3). (0.015 M solution) x (3 protons per molecule) = 0.045 N

Rank the following aqueous solutions from highest to lowest boiling point. A. 1M glucose B. 0.6M potassium chloride C. 0.5M magnesium bromide

C. 0.5M magnesium bromide -highest B. 0.6M potassium chloride A. 1M glucose- lowest The correct answer is 0.5 M magnesium bromide, followed by 0.6 M potassium chloride, followed by 0.1 M glucose. Colligative properties like boiling point elevation depend on the number of dissolved particles rather than their chemical identity. The solution with the greatest ratio of solute molecules to water molecules will have the highest boiling point. Since 0.5 M MgBr2 gives rise to 3 solute particles for every molecule of MgBr2 dissolved, the entire solution will be 1.5 M in solute particles. Similarly, a molecule of KCl generates two ions, so a 0.6 M KCl solution will be 1.2 M in solutes. Because glucose does not dissociate into ions, a 1.0 M solution of glucose will actually give rise to the fewest number of solute particles and raise the boiling point by the smallest amount.

Beta plus decay emits a beta+ particle that has A. 1e B. 2e C. 1 positron D. 2 positrons

C. 1 positron a positron is a beta+ particle

The half-life of 18F is 110 minutes. If 5 grams of FDG (181 grams/mol) remain after 5 hours and 30 minutes, how much energy was emitted from the patient's body in the form of 2 gamma rays from radioactive decay of the single FDG molecule? energy levels of the photons is 511 keV A. 3.57 x 10^3 keV B. 1.83 x 10^2 keV C. 1.19 x 10^26 keV D. 3.2 x 10^26 keV

C. 1.19 x 10^26 keV This question requires us to determine how many half-lives have occurred after 5.5 hours. 5.5 hr x (60 min/1 hr) = 330 minutes, or 3 half-lives. Working backward from the final amount of 5 g we see that we must have started with 40 g of the original sample: 5 g x 2 (first half-life) x 2 (second half-life) x 2 (third half-life) = 40 g original sample Thus 35 grams of FDG must have undergone positive beta decay. 35 grams x (1 mol/181 grams) = about 1/6 mol of FDG. For each FDG molecule that decayed, 2 gamma rays were emitted. Therefore, 1/3 moles of gamma rays were emitted. (1/3) x (6.022 x 10^23) * 511 keV = approximately 1 x 10^26 keV.

If a solutions pH was lowered by 1 pH unit, this would correspond to an increase in [H3O+] by a factor of ___ A. 1 B. 3 C. 10 D. 100

C. 10 The pH scale is logarithmic (pH = -log([H+]). A difference of 3 pH units corresponds to a 10^3 = 1000-fold difference in proton concentrations.

A tall tube is evacuated, and its stopcock closed. The open end of the tube is immersed into a container of water (density 10^3 kg/m3) that is open to the atmosphere (pressure 10^5 N/m2). When the stopcock is opened, how far up the tube will the water rise? A. 1 m B. 5 m C. 10 m D. 20 m

C. 10 m The answer to this question is C because the water will rise to a height such that the weight (mass multiplied by gravitational acceleration) of the water column equals the atmospheric pressure multiplied by the tube cross-sectional area A. Because mass is density times volume, it follows that 103 kg/m3 × h × A × 10 m/s2 = 105 N/m2 × A, where h is the height sought. Solving for h yields h = 105 N/m2/(104 N/m3) = 10 m.

How many ppm is 1%? A. 100 B. 1,000 C. 10,000 D. 100,000

C. 10,000 The abbreviation ppm stands for parts per million (parts). so the question is asking for 1% of 1,000,000

If a 3-kg rabbit's leg muscles act as imperfectly elastic springs, how much energy will they hold if the rabbit lands from a height of 0.5 m and its legs are compressed by 0.2 m? A. -0.6 J B. 0 J C. 13 J D. 14.7 J

C. 13 J Potential energy of the rabbit at the peak of its height is PE = (3)(10)(0.5) = 15 J, or, if we're more exact and use 9.81 for g, slightly less than that. 14.7 J seems about right. 13 J is clearly a little on the low side, but this is exactly what's needed for this question, since the question stem is dropping a big hint that some energy is lost.

C-O on IR spectrum A. 1650 to 1750 B. 3200 to 3500 sharp peak C. 1500 to 2000 broad peak D. 2500 to 3300 and 1750

C. 1500 to 2000 broad peak

Kw = what value A. 2.6E-5 B. 1 C. 1E-14

C. 1E-14

if given 5M and 2L of HF what is the concentration of 4L of NaOH needed to neutralize the acid A. 7/4 B. 8/5 C. 2.5 D. 2/5

C. 2.5 Ma x Va = Mb x Vb (5)(2) = (4)(X) X= 2.5 you can also use Normality instead of molarity, same exact idea

The human frequency range is from 20 Hz to 20 kHz, where 8-20 kHz is high frequency. Ultrasound frequencies must be greater than which minimum threshold? A. 1 kHz. B. 10 kHz. C. 20 kHz. D. 40 MHz.

C. 20 kHz. Ultrasound is defined as sound with a frequency above the human range of hearing ("ultra" = "above"). The human range of hearing is 20 Hz to 20 kHz, where high frequencies are 8-20 kHz, so the minimum threshold for ultrasound is 20 kHz.

functional groups with C triple bonds normally peak around which wavelength on an IR spectroscopy A. 1000-1200 B. 1600-1800 C. 2100-2300 D. 3000-3500

C. 2100-2300 medium interaction so medium transmittance

How many liters of carbon dioxide at STP are produced by reacting 100 g of calcium carbonate with an excess of hydrochloric acid? A. 0.0 L B. 11.2 L C. 22.4 L D. 44.8 L

C. 22.4 L The balanced chemical reaction is: CaCO3 (s) + 2 HCl (aq) → CO2 (g) + H2O (l) + CaCl2 (aq) From the periodic table, the formula weight of calcium carbonate is 40 + 12 + 3(16) = 100 g/mol. 100 g of calcium carbonate therefore represents one mole, and based on the reaction, this will produce one mole of carbon dioxide gas. Remember, one mole of any gas at STP (standard temperature and pressure) has a volume of 22.4 L! Thus, the volume of gas produced will be 22.4 L at STP. 100 g CaCO3 x 1 mol/100 g x 1 CO2/1 CaCO3 x 22.4 L/mol = 22.4 L CO2

alpha radiation particles consists of (select all) A. 1p B. 2e C. 2n D. 2p E. 1n

C. 2n D. 2p no e- so +2 charged

A balloon has a volume of 3.0 L at 25°C. What is the approximate volume of the balloon at 50°C? A. 1.5 L B. 2.0 L C. 3.3 L D. 6.0 L

C. 3.3 L Charles' law states that there is a direct relationship between the volume of an ideal gas and its temperature, when pressure is constant. Note that the temperature must be in Kelvin! We can approximate the initial temperature as 300 K and the final temperature as 320 K. (3.0 L)/(300 K) = V2/(320 K) [(3.0)(320)] / (300) = (3.0)(1.1) = 3.3 L

A compound with 5 different chiral centers can have ________ stereoisomers A. 5 B. 25 C. 32 D. 125

C. 32 2^5

OH alcohol on IR spectrum A. 1650 to 1750 B. 3200 to 3500 sharp peak C. 3200 to 3500 broad peak D. 2500 to 3300 and 1750

C. 3200 to 3500 broad peak

During the reduction of a mole of oxygen to water shown, how much charge is transferred? Faraday's constant can be approximated as 10^5 C/mol e- H+ + O2 = H2O A. 1 x 10^5 C B. 2 x 10^5 C C. 4 x 10^5 C D. 6 x 10^8 C

C. 4 x 10^5 C First, we need to balance the reaction: O2 (g) + 4 H+ → 2 H2O (l) This reaction is now balanced in terms of atoms, but as a redox reaction, it must also be balanced with regard to charge. We currently have a +4 charge on the left and a 0 charge on the right. To balance, we must add 4 electrons to the left side of the reaction. O2 (g) + 4 H+ + 4 e- → 2 H2O (l) Now, we can see that 4 moles of electrons are transferred per mole of O2. Faraday's constant tells us that approximately 10^5 coulombs are present per mole of electrons, so: 4 mol e- x 10^5 C/mol e- = 4 x 10^5 C

A person, whose eye has a lens-to-retina distance of 2.0 cm, can only clearly see objects that are closer than 1.0 m away. What is the strength S of the person's eye lens? (Note: Use the thin lens formula) A. -50 D B. -10 D C. 51 D D. 55 D

C. 51 D thin lens formula = S = 1/o + 1/i According to the thin lens formula, the strength of the eye lens is equal to the inverse of the focal length of the eye lens. Its numerical value is given by (1 m)^−1 + (0.02 m)^−1 = 1 D + 50 D = 51

Neutralizing a 2 L solution of 3 M acetic acid would require the addition of: A. 1.5 moles strong base B. 3 moles strong base C. 6 moles strong base

C. 6 moles strong base give the equation Ma x Va = Mb x Vb (3)(2) = ML M = mol/L mol = ML plug in mol (3)(2) = mol = 6 moles

2 R2NBCl2 + 5 LiPH2 → 4 LiCl + 3 PH3 + Intermediate What is the maximum volume of PH3(g) that a chemist can obtain from the reaction shown by the equation if 0.005 mol LiPH2 reacts with 0.002 mol R2NBCl2 at 0ºC and 1 atm? A. 0.672 mL B. 6.72 mL C. 67.2 mL D. 67.2 L

C. 67.2 mL The answer to this question relies both upon the molar ratios of the equation and on the fact that, at STP, one mole of a gas will occupy 22.4 L. The equation shows that mixing 2 moles of R2NBCl2 with 5 moles of LiPH2 will produce 3 moles of PH3. Under the conditions given in the question, 0.003 mol PH3 will be produced. At 0ºC and 1 atm (STP), this will occupy 0.003 mol × 22.4 L/mol or 67.2 mL. Thus choice C is the correct answer.

According to the data in Table 1, what mass of captopril must be dissolved in 3 L of plasma at pH 7.4 to inhibit 50% of ACE enzyme activity in vivo? Assume an equal volume of distribution. Given IC50 is 0.012 µM at pH of 4-9 A. 7.9 pg B. 7.9 ng C. 7.9 µg D. 7.9 g

C. 7.9 µg (0.012 x 10-6 mol captopril/L plasma) (3 L plasma) (220 g captopril/mol captopril) = 7.92 x 10-6 g captopril = 7.92 µg captopril Remember, 1 µg = 1 x 10-6 g.

If a 65-kg man undergoes a turning acceleration of 5 m/s^2 during a running turn, what is the magnitude of force experienced by the foot due to the ground? A. 325 B. 650 C. 750

C. 750 To turn while running, the foot must push off the ground, applying a shearing force while simultaneously supporting the weight of the body. Forces: N ground on foot, acceleration force (5m/s^2) Fnormal = mg = (65 kg)(10 m/s2) = 650 N Fturning = (65 kg)(5 m/s2) = 325 N The combined vector: round 650 N to 700 N and round 325 N to 300 N. √(300^2 + 700^2) = √(90000 + 490000) = √(580000) = √(58 x 10^4) = (√58) x 10^2 The square root of 58 falls between 7 and 8, so the overall value of our answer falls between 700 and 800, meaning that choice C must be correct (the actual value is 761).

Given the data discussed in the passage, if a hospital requires at least 250 g of 145Nd (93% of the decay occurs in the first 100 minutes) to complete a standard cardiac image, how long will it be before it must replace a 2000 g sample? A. 25min B. 50min C. 75min D. 100min

C. 75min The decay of a substance can be calculated by knowing its half-life, the time it takes for half of the original amount to decay. The final paragraph states that about 93% of the decay occurs in the first 100 minutes. This leaves about 7% of the sample left, which would indicate that close to 4 half-lives (0.5^4 = 0.0625 = 6.25% remaining) have passed at the 100-minute mark. Thus, 1 half-life = 100/4 = 25 minutes. The hospital sample is 2000 g and must be replaced once it reaches 250/2000, or 1/8 of its original radioactivity. Using the shortcut that the amount remaining = (1/2)n, where n represents the number of half-lives that have passed, we can calculate 1/8 = (1/2)3. Therefore, 3 half-lives will pass before the sample drops below 250 g. At 25 minutes apiece, 25 min (3) = 75 minutes.

the equivalence point of a weak acid/strong base titration is A. < 7 B. 7 C. > 7

C. > 7 the products are H2O, salt, and the conjugate base H2O and salt are neutral the conjugate base of weak acids is stronger so it has an effect on pH

The pH of a 1 L phosphate buffer solution was measured as 7.6, but the experimental procedure calls for a pH 7.2 buffer. Which method will adjust the solution to the proper pH? (Note: The pKa values for phosphoric acid are 2.2, 7.2, and 12.3.) A. Add enough 1 M Na2HPO3 to increase the phosphate anion concentration ten-fold. B. Add 1 M NaOH to neutralize a portion of the hydronium ions found in the solution. C. Alter the ratio of monosodium/disodium phosphate added to favor the monosodium species. D. Add 100 mL distilled, deionized water to dilute the basicity of the buffer.

C. Alter the ratio of monosodium/disodium phosphate added to favor the monosodium species. The answer to this question is C because, in order to lower the pH of a buffer, the proportion of acidic buffer component must be increased. Adding strong base, diluting with water, or adding a different basic salt will not lower the pH.

The azimuthal quantum number corresponds with which of the following? A. The potential energy of the electron B. Approximate radial size of an electron cloud C. Approximate geometric shape of the orbital D. Number of valence electrons orbiting a nucleus

C. Approximate geometric shape of the orbital this quantum number is l

Which of the four DNA bases contains the largest number of hydrogen bond acceptors when involved in a Watson-Crick base pair? A. A B. T C. C D. G

C. C The answer is B because the hydrogen bond acceptors are N and O. Adenine contains 1 donor and 1 acceptor, thymine contains 1 donor and 1 acceptor, guanine contains 2 donors and 1 acceptor, and cytosine contains 1 donor and 2 acceptors. This is a Knowledge of Scientific Concepts and Principles question because you must recall structure of a Watson-Crick base pair.

Nitrogen primarily exists in the atmosphere as a diatomic gas. Which of the following is true about this form of nitrogen? A. The presence of a lone pair of electrons on each nitrogen atom in the molecule allows it to act as a strong Lewis base. B. The triple bond of electrons creates a region of high electron density that allows N2 to be very reactive as a nucleophile. C. Diatomic nitrogen gas is relatively inert and can be used as the atmosphere in laboratory conditions to prevent unwanted side reactions. D. Atmospheric nitrogen reacts spontaneously with carbon dioxide, which keeps atmospheric CO2 levels at a relatively low 0.04% (on a molar basis) of the atmosphere.

C. Diatomic nitrogen gas is relatively inert and can be used as the atmosphere in laboratory conditions to prevent unwanted side reactions. The MCAT will expect you to be familiar with N2 as a very inert gas. It makes up approximately 80% of the air you breathe, yet has no significant chemical reactions with your lungs - or with anything other than nitrogen-fixing plants. This information implies that nitrogen is very inert (unreactive). As such, it would serve as a good artificial atmosphere when working with reagents that might react with oxygen or other gases.

what is the most likely mechanism for a steroid hormones entry into the cell? A. Active transport B. Receptor mediated endocytosis C. Diffusion directly through the membrane D. Passage through an ion channel

C. Diffusion directly through the membrane its nonpolar

During the production of insulin, the translated polypeptide is cleaved into the mature form and secreted from the cell. The cleavage most likely takes place in which of the following locations? A. Nucleus B. Ribosomes C. Endomembrane system D. Cytoplasm

C. Endomembrane system Endomembrane system is the Golgi apparatus

which of the following does work not equal A. PΔV B. power(W) x t C. F x area D. F x d

C. F x area this is the formula for pressure

In a follow-up experiment, two identical gurneys are placed side-by-side on a ramp with their wheels locked to eliminate spinning. Gurney 1 has a dummy placed on it to give it a total mass of 200 kg, while Gurney 2 is loaded with a dummy that makes it only 50 kg overall. If the ramp has a coefficient of friction of μs, which gurney is more likely to slide down the ramp? A. Gurney 1, due to the increased force of gravity B. Gurney 2, due to the reduced force of static friction C. Gurney 1 and Gurney 2 are equally likely to slide. D. Neither gurney can slide unless the wheels are unlocked.

C. Gurney 1 and Gurney 2 are equally likely to slide. In order for the gurney to slide down the ramp, the force pulling it downward (mgsinƟ) must be greater than the static frictional force (μsFN = μsmgcosƟ). The net force on each gurney is thus Fnet = (mgsinƟ) - (μsmgcosƟ). Since net force is equal to the product of mass and acceleration, we can rewrite this equation as ma = (mgsinƟ) - (μsmgcosƟ), where an acceleration greater than 0 means the gurney will slide down the ramp. The mass of the gurney is present in all terms and can be canceled, meaning that it is not a factor in whether the gurney slides. Thus, both gurneys have an equal likelihood of slipping down the ramp, regardless of the fact that they have different total masses.

What reactant would form an ester from a carboxylic acid A. ROOR, heat B. CH3Br, heat C. H2SO4, CH3OH what is this process called?

C. H2SO4, CH3OH fischer esterification

which if these reactants causes a decarboxylation reaction A. PCC B. 1)LAH 2)H3O+ C. Heat

C. Heat turns carboxylic acid into acidic acid and CO2 turn pyruvate into acetyl CoA turns THCA into THC

Phosphorous acid, a common ingredient used for potable water treatment, has a molecular formula of: A. H3PO5 B. H3PO4 C. H3PO3 D. H3PO2

C. H₃PO₃

NaOH(s) + H2O(ℓ) → NaOH(aq) this reaction occurs spontaneously and increases in temperature throughout the reaction Which of the following changes in state functions occurred during the dissolution shown in Equation 1? I. ΔH < 0 II. ΔG > 0 III. ΔS > 0 A. I only B. II only C. I and III D. I and II

C. I and III It states that the temperature increased during the preparation of the NaOH solution. Because heat is liberated, the dissolution of solid NaOH is an exothermic process for which the enthalpy change ΔH < 0. As NaOH dissolves, becomes aq, the system becomes more random (disordered), which corresponds to a positive entropy change, and ΔS > 0. The free energy change is given by ΔG = ΔH - TΔS and must be less than zero when the value of ΔH is negative and that of ΔS is positive, because the absolute temperature T is always a positive value. The reaction is also spontaneous corresponding to a -ΔG

A ray of white light moves through the air and strikes the surface of water in a beaker. The index of refraction of the water is 1.33 and the angle of incidence is 30º. All of the following are true EXCEPT: I. the angle of reflection is 30º. II. the angle of refraction is 30º. III. total internal reflection will result, depending on the critical angle. A. I only B. I and III only C. II and III only D. I, II, and III

C. II and III only Light entering a more dense medium will bend towards the normal. With an angle of incidence of 30º, the angle of refraction must be less than 30º (II). Total internal reflection can only result when a ray of light begins in a higher-index material and reaches a boundary with a lower-index one (e.g. starting in water and moving towards air). Here, the light ray started in air (n = 1) and moved into water (n ∼ 1.3), making total internal reflection impossible (III).

what agent makes this reaction occur? A. Reducing reagent B. t-butyl C. Oxidizing

C. Oxidizing Oxidizing reagents turn alcohols into aldehydes, ketones, or carboxylic acids Ex: PCC

A description of the structures of four protein Protein 1: 32 kDa monomer Protein 2: Disulfide-linked homodimer comprised of 19 kDa monomers Protein 3: Homotrimer comprised of 25 kDa monomers Protein 4: Homodimer comprised of 38 kDa monomers Which protein has the highest electrophoretic mobility in SDS-PAGE under non-reducing conditions? (Note: There are no disulfide interactions unless stated in the table.) A. Protein 1 B. Protein 2 C. Protein 3 D. Protein 4

C. Protein 3 The answer is C because in an SDS-PAGE gel that is run under non-reducing conditions, proteins 1, 3, and 4 will run as monomers. Protein 2 will run as a dimer because the disulfide bonds between Cys residues are not reduced. The running masses will then be: A = 32, B = 38, C = 25, D = 38. As the smallest one, Protein 3 will have the greatest electrophoretic mobility.

In β-minus decay a neutron is converted into a ________ and an electron is _________ A. Proton, Gained B. Electron, Lossed C. Proton, Lossed D. Electron, Gained

C. Proton, Lossed so the element will change because it gained a proton

what is a molecular formula of an amide A. RCNH3 B. RCNH2 C. RCONR2 D. RCONR3

C. RCONR2

Surfactants exert catalytic effects through which of the following mechanisms? A. Making the catalyzed reaction more energetically favorable B. Changing the equilibrium constant of the catalyzed reaction to favor the products C. Reducing the activation energy of the reaction D. Covalently transferring a reactive functional group to a reactant

C. Reducing the activation energy of the reaction this is the whole point of a catalyst

Which of the following compounds is an amphiprotic species? A. Acetic acid, HC2H3O2 B. Sodium acetate, NaC2H3O2 C. Sodium bicarbonate, NaHCO3 D. Sodium carbonate, Na2CO3

C. Sodium bicarbonate, NaHCO3 An amphiprotic species is one that can act as an acid or a base. Sodium bicarbonate dissolves in aqueous solution to produce a strong base (NaOH), which completely dissociates, and carbonic acid, a weak acid. The solution is therefore basic. The bicarbonate ion, HCO3-, has an acidic proton, so it can act as a Bronsted-Lowry acid by loss of a hydrogen ion forming carbonate, or it can act as a B-L base by accepting a hydrogen ion to form carbonic acid, H2CO3.

Squalene is produced in the human body as a precursor to which important class of compounds? (look at pic) A. Triacyl glycerols B. Phospholipids C. Steroid hormones D. Prostaglandins

C. Steroid hormones squalene makes cholesterol which is a precursor to steroid hormones these have a 4 fused ring system

Why is the velocity of blood flow slower in capillaries than in arteries? A. Capillary walls are more elastic than arterial walls. B. Capillaries have less resistance to blood flow than arteries. C. The total cross-sectional area of capillaries exceeds that of arteries. D. Blood pressure is higher in the capillaries than in arteries.

C. The total cross-sectional area of capillaries exceeds that of arteries. The high number of capillaries in the body means that the total cross-sectional area of these vessels is larger than any other vessel type in the circulatory system. This causes the velocity of the blood to decrease.

what is the ground state electron configuration of Co2+? A. [Ar]4s^2 3d^5 B. [Ar]4s^2 3d^7 C. [Ar]3d^7 D. [Ar]4s^2 4d^5

C. [Ar]3d^7 During the ionization of transition metals, electrons from 4s subshell orbitals are generally removed before those from 3d subshell orbitals. This is because it is generally true that, when occupied by electrons, electrons of 4s subshell orbitals are higher energy than those in 3d subshell orbitals.

electrochemical cell A. all human cells B. Neuron C. a device that transforms chemical energy into electrical energy D. a device that transforms electrical energy into chemical energy

C. a device that transforms chemical energy into electrical energy aka battery

the reactants acid chloride, carboxylic acid, and pyridine for what products. Select all that apply A. aldehyde B. pyridinamine C. acid anhydride

C. acid anhydride pyridine takes the H off the carboxylic acid to create a nucleophile

acid + basic solution = A. acid ionizes less B. base ionizes more C. acid ionizes more D. base ionizes less

C. acid ionizes more there is more base in the solution (basic) so the acid will dissociate more to cancel it out

in IUPAC nomenclature, the suffix of a word is A. based on the functional group attached to the lowest numbered C B. based on if an alcohol is present C. based on the most important functional group

C. based on the most important functional group

SDS-PAGE gels that run under non-reducing conditions: A. cause larger proteins to go further B. cause double stranded DNA to run as single strands C. cause dimers to run as monomers

C. cause dimers to run as monomers

What molecules absorb UV light the best during a UV spectroscopy A. alcohol B. cyclic C. conjugated

C. conjugated conjugated pi system have closer electron orbitals, the allows electrons to be more easily excited by the UV radiation

Aspartame (L-α-aspartyl-L-phenylalanine methyl ester) is a very well-known artificial sweetener found in the large majority of non-sugar containing food products. This compound is classified as a(n): A. hexose. B. phenol. C. dipeptide. D. amino acid.

C. dipeptide. Aspartame contains two amino acids as mentioned above in the formula; thus, it is a dipeptide. Aspartame is the methyl ester of the dipeptide containing aspartic acid and phenylalanine. Of course, you do not need to have any outside knowledge about the structure of aspartame, but you should be able to recognize the structure below as a dipeptide.

L-glucose and D-glucose are what to one another A. diastereomers B. anomers C. enantiomers D. epimers

C. enantiomers

addition of solute A. increases the temp that a solvent stays liquid B. decreases the temp that a solvent stays liquid C. expands the range of temp that a solvent stays liquid

C. expands the range of temp that a solvent stays liquid raises BP decreases FP

what happens if O undergoes beta minus decay A. its becomes N+ B. it becomes N- C. it becomes F+ D. it becomes F-

C. it becomes F+ beta minus decay converts 1n into a p turning O into F, it then ejects 1e- and gains a p making it positive

NMR spectroscopy determines a molecules A. light polarization of a molecule B. functional groups of a molecule, this can also help identify the molecule C. molecular formula of the molecule D. boiling point of the molecule, this can also help identify the molecule depending

C. molecular formula of the molecule it does this by determining the net nuclear spin of atoms

In the Bohr model of the atom, radiation is emitted whenever electrons: A. change orbits B. undergo acceleration C. move to orbits of lower energy D. move to orbits of a larger radius

C. move to orbits of lower energy In the Bohr model of an atom, energy is emitted only when an electron falls from a higher to a lower energy level. Thus, answer choice C is the best answer.

The typical range of human hearing spans from 20 Hz to 20,000 Hz, and the speed of sound in air is approximately 340 m/s. With this information in mind, an ultrasound signal: A. must also have a frequency between 20 Hz and 20,000 Hz. B. must have a frequency lower than 20 Hz. C. must have a wavelength lower than 0.017 m. D. must have a wavelength higher than 17 m.

C. must have a wavelength lower than 0.017 m. The prefix "ultra" means "higher," as we can discern using our knowledge that ultraviolet light is light with a frequency higher than that of visible light. Ultrasound, then, likely also has a frequency higher than the typical range heard by a human. No answer choice states this directly, so we must calculate the wavelength that this corresponds to. To do so, we can use the simple equation v = λf. 340 m/s = λ(20000 Hz) 3.4 x 10^2 m/s = λ(2 x 10^4 Hz) λ =(3.4 x 10^2 m/s) / (2 x 10^4 Hz) λ =1.7 x 10^-2 m = 0.017 m

liquid-liquid extractions separate solutions into: (select all) A. alcohols B. higher boiling point C. organic phase D. lower boiling point E. Aqueous phase

C. organic phase E. Aqueous phase

At what pH do buffers work best? A. pH that is as close to the pKa of the weak base B. pH that is as close to the pKa of the conjugate base C. pH that is as close to the pKa of the weak acid

C. pH that is as close to the pKa of the weak acid the pKa of the henderson hasselbalch equation is from the weak acid pH = pKa + log [A-]/[HA]

To determine a protein's thermodynamic stability, chemical denaturation studies can be performed. Assuming that only the native and unfolded states can be observed under experimentally available conditions, what is the most likely shape of the curve for the dependence of the fraction of folded protein upon denaturant concentration? A. hyperbolic B. linear C. sigmoidal D. exponential

C. sigmoidal The answer is C because the unfolding of proteins is a cooperative process. Cooperative processes are marked by sigmoidal curves.

disulfide bonds at an important bond in which protein structure A. primary B. secondary C. tertiary D. quaternary

C. tertiary

binding energy A. the amount of energy needed to break a bond between atoms B. the amount of energy needed to create a bond between atoms C. the amount of energy needed to break up the nucleus of an atom

C. the amount of energy needed to break up the nucleus of an atom smallest amount of energy required to remove a particle from a system of particles

intensity of light corresponds to A. the frequency B. the wavelength C. the number of photons in the light

C. the number of photons in the light

Na2CO3 reacts with 10H2O to produce a colorless solution If red litmus paper is dipped into the Na2CO3 solution, it will: A. remain red because carbonate is an acidic salt B. remain red because sodium carbonate is neutral C. turn blue because carbonate reacts with water do produce OH- D. turn blue because sodium ions because NaOH in water

C. turn blue because carbonate reacts with water do produce OH- In water, carbonate will undergo the following reaction: CO3 2-(aq) + H2O(l) → HCO3 -(aq) + OH-(aq). Red litmus paper will turn blue in a base. Thus, C is the best answer.

Kw means A. equilibrium of energy B. equilibrium of work C. equilibrium water autoionization

C. water autoionization water reacts with itself to form ions (H3O+, OH-)

in what condition will NMR spectroscopy detect a net nuclear spin of an atom A. when the protons = neutrons B. when protons = e- C. when protons are different from neutrons D. when protons are different from e-

C. when protons are different from neutrons

what phosphate of ATP is transferred A. α32P-ATP B. β32P-ATP C. γ32P-ATP D. δ32P-ATP

C. γ32P-ATP gamma phosphate is the third phosphate of ATP

Light inside the thin glass tube of a laparoscopic surgical device strikes the edge of the glass tube and is entirely reflected back into the tube, with none of the light exiting to the surrounding medium. Which of the following must be true? A. θincident = 90º B. θincident = 0º C. θincident ≥ θcritical D. θrefracted = θincident

C. θincident ≥ θcritical The question describes total internal reflection, when a light ray bounces inside a medium with a higher refractive index than the surrounding medium. For a light ray to totally internally reflect, rather than exit and refract (bend), the light ray must strike the edge of the glass tube at an angle equal or greater than the critical angle.

draw enamine

C=C with a N attached to one C and 2 R groups Hint: En (comes from alkENe i.e. C C double bond) amine (i.e. the amine group attached)

draw amide

C=O bond with N attached to the C and 2 other groups

what alkaline earth metals make strong bases

Ca, Sr, Ba Ca(OH)2 Sr(OH)2 Ba(OH)2

boiling point elevation or freezing point depression equation

Change in T = K(b or f)im K(b or f): boiling point elevation or freezing point depression constant (will be given) i: Van't Hoff factor = number of particles after ionization -Ex: NaCl has an i = 2 because Na+ and Cl- m: molality = mol solute/ Kg solvent - for water 1 Kg ~ 1 L

draw an aldol

Contains both aldehyde and alcohol functional groups.

In the D/L system used to classify amino acids is the higher priority group for a D amino acid on the left or the right of the chiral carbon

D is on the right L is on the left

If the average pitcher is releasing the ball from a height of 1.8 m above the ground, and the pitcher's mound is 0.2 m higher than the rest of the baseball field, at what height would the catcher need to hold his glove to catch the pitched ball? (Note: neglect air resistance, estimate the acceleration due to gravity as 10 m/s2, and assume the pitcher is throwing the ball only in the horizontal direction.) the velocity of each ball was a constant 30m/s as it crossed home plate 18 meters away. A. 2.0 m above the ground B. 1.8 m above the ground C. 0.5 m above the ground D. 0.2 m above the ground

D. 0.2 m above the ground (18 m) / (30 m/s) = (18/30) s = 3/5 s = 0.6 s The ball is released from a position 2 m off the ground (0.2 m from the pitcher's mound and 1.8 m from the pitcher). To calculate the distance the ball falls during 0.6 s, we can use the equation d = v0t + 1/2at²: d = (0 m/s)(0.6 s) + 1/2(10 m/s2)(0.6 s)² d = 1/2(10)(0.36) = 1/2(3.6) = 1.8 The ball has fallen 1.8 m from an initial height of 2.0 m. Thus, the catcher must hold the glove 0.2 m above the ground to catch the pitch.

GK is a homodimer, how many bands would be observable if GK was subjected to native PAGE and SDS-PAGE analysis? (Note: Assume GK maintains a single state in solution.) A. 2 bands for both B. 1 for native PAGE and 2 for SDS-PAGE C. 2 for native PAGEand 1 for SDS-PAGE D. 1 band for both

D. 1 band for both The answer is D because GK is a homodimer, which means that both preservation and disruption of the quaternary structure results in a single gel band. This is because the subunits of the dimer are the same molecular weight. Therefore, both electrophoresis experiments should result in a single gel band. This is a Reasoning about the Design and Execution of Research question because you must understand the differences in the experimental design of two types of electrophoresis and how those differences affect the protein.

What is the pH of a 0.010 M sodium hydroxide solution at 25°C? A. 1 B. 2 C. 7 D. 12

D. 12 DONT BE A ****ING ******** BITCH sodium HYDROXIDE obviously is going to be basic Although you can start by calculating the pH, you should immediately notice that choice D is the only option that is basic. To answer this question using math, first note that sodium hydroxide is a strong base that completely dissociates in aqueous solution. Therefore, the hydroxide ion concentration of this solution is also 0.010 M, or (in scientific notation) 1 E-2 M. Taking the negative logarithm of the hydroxide concentration gives a pOH of 2. Since -Log (1E-2) = 2 Since pH + pOH = 14 (at 25ºC), we can calculate the pH by subtracting the pOH from 14, which yields pH = 14 - 2 = 12.

How many grams of hydrogen gas are required to completely react with 32 g of oxygen gas to form hydrogen peroxide? A. 0.5 g B. 1.0 g C. 1.5 g D. 2.0 g

D. 2.0 g The formation reaction for hydrogen peroxide is: H2 (g) + O2 (g) → H2O2 (l) Moving on to the stoichiometry calculation: 32 g O2 x (1 mol/32 g) x (1 H2/1 O2) x 2 g/mol = 2 g H2

A glass rod is rubbed with a silk scarf producing a charge of +3.2 × 10^-9 C on the rod. (Recall that the magnitude of the proton and electron charges is 1.6 × 10^-19 C.) The glass rod has: A. 5.1 × 10^11 protons added to it. B. 5.1 × 10^11 electrons removed from it. C. 2.0 × 10^10 protons added to it. D. 2.0 × 10^10 electrons removed from it.

D. 2.0 × 10^10 electrons removed from it. The answer to this question is D because the number of charges in excess can be computed as +3.2E -9 C/1.6E-19 C = +2.0 E 10. This means that the rod has an excess of positive charge, created by removing a number of +2.0E 10 electrons from the material, as it is not possible to add protons in a manner described in this question.

What is the molar solubility of ferrous (II) hydroxide in water at 25°C?The passage states that the Ksp of ferrous (II) hydroxide is 3.2 x 10^-14. A. 2.1 x 10^-8 B. 8.2 x 10^-8 C. 2.8 x 10^-6 D. 2.1 x 10^-5

D. 2.1 x 10-5 When Ksp is known, we can determine molar solubility from the dissociation reaction. Fe(OH)2 (aq) + H2O (l) → Fe2+(aq) + 2 OH- (aq) Note that Fe(OH)2 dissociates into three ions (one Fe2+ and 2 OH-). Given this 2:1 ratio, Ksp = [Fe2+][OH-]^2 = [x][2x]^2 = 4x^3, where x represents the molar solubility. Next, we must divide Ksp by 4, then take its cube root to solve for x. Dividing 3.2 by 4 is more difficult than dividing 32 by 4, so we can manipulate scientific notation and rewrite Ksp in an easier format. Ksp = 4x^3 = 3.2 x 10^-14 = 32 x 10^-15 8 x 10-15 = x^3 2 x 10-5 = x

Which of the following is most likely to undergo positive beta decay? A. 14C B. 13C C. 17O D. 22Na

D. 22Na 22 is the weight of Na Positive beta decay, also known as positron emission, occurs when the proton-to-neutron ratio is too high. Thus, we are looking for the answer choice with the highest proton-to-neutron ratio. Of the options given, 22Na has the highest ratio, as it has 11 protons and 11 neutrons (1:1). Because choice D has the highest proton-to-neutron ratio among the answer choices, it is the most likely to undergo positive beta decay.

OH in carboxylic acid on IR spectrum A. 1650 to 1750 B. 3200 to 3500 sharp peak C. 3200 to 3500 broad peak D. 2500 to 3300 and 1750

D. 2500 to 3300 and 1750 very broad

functional groups with OH/NH bonds normally peak around which wavelength on an IR spectroscopy A. 1000-1200 B. 1600-1800 C. 2100-2300 D. 3000-3500

D. 3000-3500 highly interactive with H bonds so high transmittance

An object with a mass of 10 kg is rolled down a frictionless ramp from a height of 3 meters. If a factory worker at the bottom of the ramp slows the object until it comes to a stop, how much work must the factory worker have done? (Assume g = 10 m/s2.) A. 30 N B. 300 N C. 30J D. 300J

D. 300J You can solve this question using conservation of energy and the work-energy theorem. The object has 300 J of gravitational potential energy at the top of the ramp (U = mgh = 10 x 10 x 3). That energy is converted into kinetic energy at the bottom of the ramp. The work-energy theorem tells us that Wnet = ΔKE. Since the object is coming to rest, its ΔKE = 300 J. Thus, the work done must also equal 300 J.

A sparingly soluble metal hydroxide, M(OH)2 has a molar solubility of S mol/L at 25°C. Its Ksp value is: A. S^2 B. 2S^2 C. 2S^3 D. 4S^2

D. 4S^2 The Ksp for a substance, AaBb, equals [A]^a [B]^b. The Ksp for M(OH)2 = [M][OH-]2. If the solubility of M(OH)2 is S mol/L, then [M] = S mol/L and [OH-] = 2S mol/L. The Ksp = S(2S)^2 = S(4S^2) = 4S^3. Thus, D is the best answer.

An artificial heart valve was tested for its ability to function under extreme conditions, to a maximum flow rate of 4.00 x 10-4 m3/s. What speed would this correspond to for an average red blood cell within a blood vessel of cross-sectional area 5.00 x 10-6 m2? A. 2.00 x 10^-9 m/s B. 1.25 x 10^-2 m/s C. 4.05 x 10^-4 m/s D. 8.00 x 10^1 m/s

D. 8.00 x 10^1 m/s Watch your units! We're given one number in m3/s and another in m2. To get an answer in the units of m/s, just divide the two numbers in the question: Speed = (4 x 10-4 m3/s) / (5 x 10-6 m2) = (4/5) x (10-4/10-6) = 0.8 x 102 m/s = 8 x 101 m/s This can also be solved using the equation flow rate = cross-sectional area x velocity.

What is the average power consumed by a 64-year-old woman who takes 27s to climb up 30 steps during the ascent of the 15-cm-high steps, if her mass is 54 kg? A. 10 W B. 20 W C. 40 W D. 90 W

D. 90 W The answer to this question is D because the power consumed is P = PE /time = mgh/t. From Table 1, there are 30 steps and t = 27 s. Then P = (54 kg × 10 m/s2 × 30 steps × 0.15 m/step) / (27s) = 90 W.

While the composition of oxygen and nitrogen in air does not change with altitude, the decreasing temperature at high altitude does change the percent of air that is composed of H2O. Assuming constant relative humidity, which of the following can be asserted about the total grams of H2O in a given volume of air at 3000 m above sea level versus at sea level? A. Assuming constant relative humidity means that air has roughly the same mass of H2O per unit volume at 3000 m above sea level. B. Whether air at very high altitude has more or less mass of H2O per unit volume than it does at sea level depends on the temperature at high altitude. C. Air has significantly more mass of H2O per unit volume at 3000 m above sea level. D. Air has significantly less mass of H2O per unit volume at 3000 m above sea level.

D. Air has significantly less mass of H2O per unit volume at 3000 m above sea level. With decreasing temperature, air is able to hold less H2O. Since colder air can hold less total H2O, this means the same relative humidity would result in less total water in the air. For example, if warm air could hold 100 g of water in a given volume, then 50% relative humidity would be 50 g of water in the air. And if cold air could only hold 40 g of water in the same given volume, then 50% relative humidity would be 20 g of water in the air. So with the same relative humidity, the cold high-altitude air has less mass of water.

Which of the following gases would decrease the pH when dissolved in an aqueous solution? A. Ammonia B. Oxygen C. Nitrogen D. Carbon dioxide

D. Carbon dioxide think about blood and bicarbonate

The reduction of ketone groups, such as those present on CoQ, produces alcohols. In contrast, oxidation of aldehydes produces which functional group? A. None of these; aldehydes cannot be oxidized further. B. Primary alcohols C. Ethers D. Carboxylic acids

D. Carboxylic acids In organic chemistry, oxidation can be conceptualized as the gain of bonds to oxygen (or the loss of bonds to hydrogen). An aldehyde, which contains a C=O double bond, can gain another bond between carbon and oxygen if it is converted to a carboxylic acid.

Which of the following energy conversions best describes what takes place in a battery-powered resistive circuit when the current is flowing? A. Electric to thermal to chemical B. Chemical to thermal to electric C. Electric to chemical to thermal D. Chemical to electric to thermal

D. Chemical to electric to thermal

Which of the following best describes the bonds between Cu2+ and the nitrogen atoms of the ammonia molecules in [Cu(NH3)4]2+? A. Ionic B. Covalent C. Coordinate ionic D. Coordinate covalent

D. Coordinate covalent The answer to this question is D because the Lewis acid-base interaction between a metal cation and an electron pair donor is known as a coordinate covalent bond. It is a Knowledge of Scientific Concepts and Principles question because you must recognize a correct scientific principle. Coordinate ionic bond does not exist so it doesn't matter that its a metal and nonmetal

interaction between a metal cation and an electron pair donor is called A. Ionic bond B. Covalent bond C. Coordinate ionic bond D. Coordinate covalent bond

D. Coordinate covalent bond

the stereoisomer of a carbohydrate with the hydroxyl group pointing to the right of the highest number carbon has a configuration of A. S B. R C. L D. D

D. D to the right would be a D-glucose to the left would be an L-glucose

Which of the following proteins is responsible for the retrograde transport of organelles in cells? A. Kinesin B. Myosin C. Selectin D. Dynein

D. Dynein Dynein is a motor protein that walks towards the minus end of the microtubules, which is oriented towards the center of the cell. Movement toward the center of the cell is described as retrograde.

which of the following is not a strong acid A. HI B. HBr C. HCl D. HF

D. HF this is a weak acid

The relative thermodynamic stability of isomeric organic compounds can be inferred from which of the following types of experimental data? A. Boiling points B. UV-visible absorption spectra C. Mass spectroscopic fragmentation patterns D. Heats of combustion

D. Heats of combustion The answer to this question is D because the relative thermodynamic stability of isomers can be determined based on the amount of heat produced when the compounds are combusted; less heat, greater stability.

Thermal denaturation experiments can be used to follow the transition of double-stranded DNA into single-stranded DNA. Which of the following parameters affects the Tm of dsDNA in this experiment? I. pH of solution II. Ionic strength of solution III. Length of DNA strands A. I and II only B. I and III only C. II and III only D. I, II, and III

D. I, II, and III The answer is D because all of these parameters would affect the thermodynamic stability of the DNA double helix. Significant drops in pH would result in protonation of hydrogen-bond acceptors, leading to a loss in base-pairing interactions. The presence of positive ions in solution (particularly Mg2+) leads to stabilization of the DNA fold via shielding of the repulsion between phosphate groups within the DNA backbone. The length of DNA strands would also play a role. Longer DNA strands are held together by more hydrogen bonds, meaning that more energy is required to denature the double-stranded DNA.

Measurements are made in two arteries with the same diameter and the same flow rate. One artery is in the head and the other is in the leg. In a person standing upright, which of the following statements is true? A. The speed of the blood in the head is greater than the speed of the blood in the leg. B. The speed of the blood in the leg is greater than the speed of the blood in the head. C. The pressure of the blood in the head is greater than the pressure of the blood in the leg. D. The pressure of the blood in the leg is greater than the pressure of the blood in the head.

D. The pressure of the blood in the leg is greater than the pressure of the blood in the head. The gravitational potential energy of the blood in the head is greater than the gravitational potential energy of the blood in the leg. As a result, the blood in the leg will have a greater pressure due to the conversion of the gravitational potential energy into the kinetic energy of molecular motion. The increased pressure is the result of the increased number of collisions between the molecules in the blood of the leg and the surface of the blood vessel. The question states that the two blood vessels have the same volume flow rate and diameter, hence the same speed.

Which experimental condition is NOT necessary to achieve reliable data for Michaelis-Menten enzyme kinetics? A. Initial velocity is measured under steady state conditions. B. Solution pH remains constant at all substrate concentrations. C. The concentration of enzyme is lower than that of substrate. D. The reaction is allowed to reach equilibrium before measurements are taken.

D. The reaction is allowed to reach equilibrium before measurements are taken. The answer to this question is D because once the reaction reaches equilibrium, measurement of Vo will be impossible and the kinetic data will look the same regardless of substrate concentration. Hence, response D is not necessary (nor desirable) to achieve reliable data for Michaelis−Menten enzyme kinetics. In contrast, Distractors A−C are essential to obtain reliable Vo versus substrate concentration data to calculate KM and Vmax using Michaelis−Menten enzyme kinetics.

What kind of image is formed by the lenses of the glasses worn by a 68-year-old male with a focal length of −50cm who sees an object 2 m away? A. Real and enlarged B. Real and reduced C. Virtual and enlarged D. Virtual and reduced

D. Virtual and reduced use thin lens equation 1/f = 1/o + 1/i 1/-0.05 = 1/2 + 1/i i must be - (virtual) to make f - i must be small to make f such a small value M = -i/o = -0.05/2 = very small number < 1 which is reduced

Pb2+(aq) + 2e- → Pb(s) E°red = -0.127 V Cu2+(aq) + 2e- → Cu(s) E°red = +0.339 V An electrolytic cell is set up in which solid Pb and solid Cu are placed in solutions, Pb is placed in a solution containing Pb2+ and Cu is placed in a solution containing Cu2+. Will an oxidation-reduction reaction occur spontaneously? A. No; E°cell = -0.212. B. No; E°cell = +0.212. C. Yes; E°cell = -0.466. D. Yes; E°cell = +0.466.

D. Yes; E°cell = +0.466. A spontaneous reaction occurs when E° is greater than zero, and this occurs if the oxidation of Pb(s) is combined with the reduction of Cu2+(aq), resulting in a net E° of +0.466 V.

A stock solution of the enzyme was prepared by diluting 0.100 mL of the commercial preparation to 25.0 mL in the buffer solution. Experiments were initiated by mixing 1.0 mL of each substrate solution with 1.0 mL of the enzyme solution. If [E]t was the concentration of lactase in the kinetics trials, what expression gives the concentration of lactase in the commercial preparation of this enzyme? A. [E]t x 25 B. [E]t x 50 C. [E]t x 200 D. [E]t x 500

D. [E]t x 500 The answer to this question is D because the commercial preparation was first diluted by 1 → 250 to prepare the stock solution, and it was further diluted by 1 → 2 by mixing with the substrate stock solution to perform the kinetics experiments. 1/250 x 1/2 = 1/500

A large dip in an IR spectroscopy graph signifies: A. a large wavelength B. a small wavelength C. a large transmittance D. a small transmittance

D. a small transmittance this means the bond absorbed a lot of the radiation and there was little transmitted through the sample

The lone pair of electrons in ammonia allows the molecule to: A. assume a planar structure. B. act as an oxidizing agent. C. act as a Lewis acid in water. D. act as a Lewis base in water.

D. act as a Lewis base in water. lewis base is a electron-pair donor

Enzymes alter the rate of chemical reactions by all of the following methods EXCEPT: A. co-localizing substrates. B. altering local pH. C. altering substrate shape. D. altering substrate primary structure.

D. altering substrate primary structure. Altering the primary structure of the substrate is not a way for the enzyme to modify the rate of chemical reaction.

base + basic solution = A. acid ionizes less B. base ionizes more C. acid ionizes more D. base ionizes less

D. base ionizes less there is already more base in the solution so the base will not need to dissociate as much

gamma rays change the _______ of an atom (select all) A. atomic number B. atomic mass C. charge D. energy

D. energy

D-glucose and D-galactose are A. stereoisomers B. anomers C. enantiomers D. epimers

D. epimers Diastereomers that differ at only one chiral center.

Mass spectroscopy determines a molecules A. light polarization of a molecule B. functional groups of a molecule, this can also help identify the molecule C. molecular formula of the molecule D. mass

D. mass OBVI

Free radicals from ionizing radiation are highly unstable and have carcinogenic effects. These effects are most likely result from damage to: A. proteins. B. phospholipids. C. the Golgi apparatus. D. nucleic acids.

D. nucleic acids. The question stem states that free radicals can cause cancer, which is a result of poorly regulated cell growth and division. Cancer is generally a product of mutations in DNA that disrupt these processes. Therefore, we must choose the answer that targets DNA.

what is the inverse of conductivity A. current B. voltage C. resistance D. resistivity

D. resistivity

What type of isomer is shown in the pic A. functional isomers B. enantiomers C. stereoisomers D. tautomers

D. tautomers Isomers can interconvert by exchanging the location of a proton. Exist in equilibrium type of constitutional isomer

How does UV spectroscopy work to identify the functional groups in a molecule A. certain wavelengths of radiation are absorbed by the functional groups, the light not transmitted is transmitted through the sample to a sensor that correlates the functional group B. radiation is absorbed by the groups and the wavelength not absorbed give off a certain color telling what the group is C. radiation is absorbed by the groups and the frequency of the wavelength absorbed is measured and correlated with a functional group D. wavelengths of radiation hit the compound, causing the electrons in the compound to be excited and absorb specific wavelengths of light

D. wavelengths of radiation hit the compound, causing the electrons in the compound to be excited and absorb specific wavelengths of light

equation that relates energy of the photon to its frequency

E = hf h- Planck's constant

The opposite of a constitutional isomer is a A. diastereomers B. anomers C. enantiomers D. epimers E. stereoisomer

E. stereoisomer same connectivity but different spatial arrangement

Energy of a photon equation

E=hf or λ = hc/E h is planks constant f is frequency

binding energy equation

E=mc^2 E- binding energy m- mass defect (the difference between the estimated mass of an atom based on the mass of molecules in the atom vs the actual mass of an atom)

Nernst equation

Ecell= E°cell - (RT/nF) (lnQ) n-moles of e- transferred F- constant = 1E5 a mathematical relationship used to calculate an ionic equilibrium potential

Electrophile vs. Nucleophile

Electrophile: electron poor, + charged, donate electrons Nucleophile: electron rich, - charged, receive electrons

match each word with its definition: Enantiomers, diastereomers, epimer A. Are NOT mirror images, will have MULTIPLE chiral centers and have different chemical AND physical properties B. A subtype of diastereomers that differ in absolute configuration at exactly one chiral carbon C. non-superimposable mirror images of each other with identical physical AND chemical properties

Enantiomers: C. non-superimposable mirror images of each other with identical physical AND chemical properties Diastereomers: A. Are NOT mirror images, will have MULTIPLE chiral centers and have different chemical AND physical properties Epimer: B. A subtype of diastereomers that differ in absolute configuration at exactly one chiral carbon Ex: the diastereomers shown in the pic, one is R, R and the other is R, S. More useful in compound with more than 2 chiral centers

Endothermic vs. Exothermic

Endo: Heat absorbed H>0 Exo: Heat released H<0

equation for force of magnetic field

F=qvBsinθ q- charge v- velocity B- magnetic field

True or false: adding a catalyst to a reaction shifts to the right producing more product

FALSE: catalysts change the speed of a reaction, not the equilibrium

True or false: Keq is not temperature dependent

False Keq is temperature dependent think heat increases enzyme activity and also the speed that molecule hit one another changing temp can cause a reaction to be product or reactant favored causing a change in Keq

True or false: Keq changes when the concentration of a reactant or produce increases

False Keq is not concentration dependent Ex: If the concentration of products is increased, the concentration of reactants will change to meet the same Keq

True or false: The faster a reaction reaches equilibrium, the more negative its ΔG value is.

False Remember that the speed of a reaction is related to its kinetic properties, not its thermodynamic properties. Gibbs free energy is a thermodynamic value and has no bearing on the speed of a reaction. A reaction with immensely more products than reactants at equilibrium will be very spontaneous, regardless of how long it takes to reach that point.

True or false: spontaneous reactions occur more quickly than non spontaneous reactions

False The reaction rate is independent of spontaneity. The spontaneity of a reaction is only dependent on the change in free energy between the products and the reactants. The reaction rate is dependent on the stability of intermediates in the reaction and activation energies.

True or false: a chiral molecule with 3 chiral center has 16 possible stereoisomers

False: 2^3= 8 so it has 8 possible stereoisomers

True or false: bonds formed have a +∆H

False: Bonds formed have a -∆H, forming bonds releases energy bonds broken have a +∆H, Bonds broken requires energy

True or false: Le Chatelier's Principle states that if a stress is applied to a system at equilibrium, it will not shift

False: States that if a stress (temp, concentration, enzyme catalyst change) is applied to a system at equilibrium, the system shifts in the direction that relieves the stress.

True or false: the suffix -diol means 2 amine groups

False: 2 alcohol groups

True or false: a heterocycle is a cyclic molecule with a odd number of atoms

False: A heterocycle is a ring of atoms of more than one kind

True or false: lactam is a cyclic molecule with a carboxyl group

False: A lactam is a cyclic amide; named according to the number carbon atoms other than the carbonyl carbon. alpha, beta, gamma, etc Beta lactam are 4 membered rings, very reactive bs of ring strain

True or false: PCC is a strong oxidizing agent so it will oxidize a primary alcohol all the way to the carboxylic acid what are other strong oxidizing agents?

False: PCC is an oxidizing agent so it will take a primary alcohol to a aldehyde but it is not strong enough to turn it into a carboxylic acid CrO3, Na2Cr2O7, Kr2Cr2O7 are strong oxidizing agents

True or false: alkyl groups (CH3) added to an molecule have an inductive effect, pulling the electrons away and stabilizing the conjugate base of an acid, making the acid stronger

False: alkyl groups (CH3) have the opposite of an inductive effect, they are inductive donors, pushing the electrons towards the negative charge, destabilizing the conjugate base and making the acid weaker electronegative atoms (Br, Cl, F) added to an molecule have an inductive effect, pulling the electrons away from a negative charged atom and stabilizing the conjugate base of an acid, making the acid stronger

True or false: gasses have lower entropy than solids

False: entropy is the disorder of a system gasses have more disorder than solids

True or false: the maillard reaction causes ice to be slippery

False: it causes food to turn brown when cooked, like steak

True or false: a solution is only a homogeneous liquid mixture

False: it doesn't have to be liquid Ex: the earth atmosphere is a homogeneous mixture

True or false: a positron is a particle that has the same charge as an electron, but has a different mass, and is emitted from the nucleus during some kinds of radioactive decay

False: it is a particle that has the same MASS as an electron, but has a POSITIVE charge, and is emitted from the nucleus during some kinds of radioactive decay

True or false: mass defect is the difference between the number of protons in an atom and the number of electrons in an atom

False: it is the difference between the estimated mass of an atom based on the mass of molecules in the atom vs the actual mass of an atom

True or false: The axial position on the chair conformation is the one sticking straight up or down and it is the most stable with bulky substituents

False: it is the one sticking straight up or down but it is the least stable the flatter equatorial position is more stable

True or false: osmotic pressure is the pressure that must be applied to cause osmotic movement across a selectively permeable membrane

False: it is the pressure that must be applied to PREVENT osmotic movement across a selectively permeable membrane counteracts the flow of osmosis

True or false: the OH in an alpha anomer is pointing up

False: look at pic

True or false: in beta plus decay, atomic mass increases by 1

False: neither type of beta decay changes atomic mass because a P becomes a N or visa versa so there is the same amount of mass

true or false: all chemical bonds appear on an IR spectrum

False: normally only polar covalent bond do, nonpolar bonds will not ex: OH, C=O, C-O

True or false: nucleophilic addition involves a nucleophile that attacks an electrophile and only kicks off a hydrogen in the process

False: nucleophilic addition doesnt involve kicking off any groups it just attacks a carbon double bonded to an oxygen

True or false: tautomers and resonance structures are the same

False: resonance structures use electron delocalization, only the bonds are moved. Tautomers move a hydrogen.

True or false: liquid distillations normally obtain a pure solution after the first distillation

False: some of the other solution is likely to have evaporated as well so multiple distillations are used often

True or false: this conformation shown in the pic is a staggered conformation

False: staggered is when the substituents are far apart this is eclipsed

true or false: chiral molecules need 4 different substituents attached

False: that is a chiral center Chiral molecules are molecules that are mirror images of each other

True or false: a nonvolatile solute readily escapes into the vapor phase.

False: that is a volatile solute, a nonvolatile solute does not readily escape into the vapor phase hence NON

true or false: an amphiprotic species is a molecule that has a polar and nonpolar region

False: that is an amphipathic molecule. amphiprotic means can act as an acid or a base.

True or false: the D/L configuration of a carb is based off of the d/l system for how that light is rotated when polarized

False: the D/L system for carbs has nothing to do with the d/l system for polarized light rotation

True or false: the d/l nomenclature system can be determined by ranking their atomic mass and seeing if it follows the d or l configuration

False: the d/l nomenclature system can only be determined by experimentation and is not associated with the R and S absolute configuration system

True or false: the double bond formed in elimination reactions occurs on both sides of the lone pair equally Bonus: whats the exception

False: the double bond will more on the more substituted side of the lone pair However, this will not occur with a LARGE BULKY BASE this will cause the least substituted double bond

true or false: the reactants and products of a galvanic cell would be reversed in an electrolytic cell but the Ecell (cell potential) would stay the same

False: the first statement is true, the reactants and products of a galvanic cell would be reversed in an electrolytic cell however if the Ecell for a galvanic cell was 2.5, the E cell for a electrolytic cell would be -2.5

true or false: tautomers are resonance structures

False: these are two similar structures formed by the breaking and forming of H bond

True or false: Immiscible fluids are solvents that are similar in boiling point

False: these are two solutions that dont mix

True or false: bronsted lowry acid base is about electrons where lewis acid base definition is about protons

False: they are switched bronsted lowry: protons Lewis: electrons

True or false: chaotropic agents break disulfide bonds of proteins

False: they break hydrogen bonds denaturing the protein

true or false: double bonds can act as electrophiles

False: they can act as nucleophiles

True or false: carbohydrates with 5-6 carbons normally exists in their linear structures in nature

False: they normally react with themselves to create a cyclic structure

true or false: True or false: the half equivalence point is the point at which the acid and base used in a titration are neutralized

False: this defines the equivalence point. The half equivalence point is the point where half of the titrant needed to reach the equivalence point has been added. half of the original acid/base has been neutralized

True or False: redox reactions describe a transfer of protons

False: transfer of electrons, not protons

True or false: when determining the D/L configuration for a carbohydrate with many different stereocenters, we look at the first chiral carbon

False: you look at the highest numbered chiral carbon Ex: the chiral center of the red hydroxyl

Iron cations

Ferrous: Fe2+ Ferric: Fe3+

Acid Catalyzed keto-enol tautomerization

First acid protonates the oxygen alpha hydrogen is removed via water and bond is moved between alpha carbon and carbonyl C C=O double bond is pushed up as a lone pair on the O

match each Fusion, deposition, sublimation A. Gas to solid B. melting C. Solid to gas

Fusion: B. melting Deposition: A. Gas to solid Sublimation: C. Solid to gas

when using the equation ΔG = ΔH - TΔS to determine the T of spontaneity, if given H and S to solve for T what will G be BONUS: at what temp will the reaction be spontaneous with a ΔH of 2000J and a ΔS of 100

G will be set to 0, the tipping point between spontaneous the nonspontaneous Bonus: ΔG = ΔH - TΔS 0 = 2000 - T (100) T = 20 K at G = 0 so <20 K the reaction is nonspontaneous and >20K the reaction is spontaneous Plug in a - value for G and see if the temp increases or decreases

what molecule is this

Glycerol

give oxidation numbers to each atom in a carboxyl group

H is +1 O is -2, there are two so -4 this totals to -3 therefore the carboxyl carbon is +3 to give a total of 0 for the neutral molecule

Sulfuric Acid

H2SO4 (Strong Acid)

CO3 2-(aq) + H2O(l) →

HCO3 -(aq) + OH-(aq). forms OH-

Chloric Acid

HClO3 (Strong Acid)

Perchloric Acid

HClO4 (Strong Acid)

Nitric acid

HNO3 (Strong Acid)

besides HI, HBr, and HCl, what are the last 4 strong acids

HNO3: Nitric acid HClO3: Chloric acid HSO4: Sulfuric acid HClO4: Perchloric acid the ones first in alphabetical order via their name have a 3 the ones second in order have a 4

alkyne suffix and definition

Have at least one carbon carbon triple bond -yne

which of the halides form strong acids?

I, Br, Cl HI, HBr, and HCl are all strong acids

Which of the following reagents is/are likely to be used to form a buffer solution with a pH of 4.3? I. A polyprotic, weak acid II. Na2HPO4 III. NH3

I. A polyprotic, weak acid II. Na2HPO4 Buffer solutions, which are intended to resist changes in pH, consist of a weak acid or weak base and its corresponding salt. The passage states that the buffer solution is acidic (pH = 3.8), so we can assume that a base is not present in the solution. Therefore, any Roman numeral (RN) that is a weak acid or a salt formed from a weak acid is a potential component of the buffer mentioned in the passage. In RN questions, we should begin by looking for the RN that appears most often in the answer choices and discount those choices that contain an answer that is obviously wrong. RN I and II are in 3 choices, so we can begin with either of those, and we can discount any answer that has III, a weak base. A polyprotic, weak acid fits the criteria for an acidic buffer solution (I). Additionally, Na2HPO4 (phosphoric acid) is a salt formed from a weak acid and would therefore be an appropriate choice as a buffer (II) NH3 is basic and would have a pH above 7

what group is pictured

Imide R1 can be a H

specific heat capacity units

J/gºC q/mT = m

kinetic energy of electron emitted after hit by a photon equation

KE = hf - work function hf- energy of photon work function- energy used to eject the electron the rest of the energy not used will be converted into KE

catalytic efficiency equation

Kcat/Km Kcat: enzyme reaction rate Km: substrate concentration

match the type of molecule with its IUPAC naming suffix Ketone, aldehyde A. -al B. -one C. -ane

Ketone: B. -one Aldehyde: A. -al

Ksp of this reaction NaCl(s) + H2O(l) <==> Na+(aq) + Cl-(aq)

Ksp = [Na+][Cl-] solids and liquids are not included, follows the same rules as Keq

Kw equation

Kw = [H+][OH-] = 1E-14 since H2O is liquid it is not in this equation as the reactant

draw the flow of fluid in laminar flow with arrows in a pipe

Laminar flow is due to shear forces (friction) between the fluid and the solid surface of the tube. This results in layers having a gradient of velocities, in which the flow is the fastest in the middle of the tube (where friction is low) and slowest near the surface (where friction is high).

E1 mechanism

Leaving group leaves making a carbocation bronsted lowrey base removes another hydrogen leaving the bond instead of attaching the carbocation the leftover lone pair then form a double bond forms the double bond with the carbocation

what earth metals make strong bases

Li, Na, K LiOH NaOH KOH

what are the 8 Strong bases

LiOH, NaOH, KOH, RbOH, CsOH earth metals Ca(OH)2, Sr(OH)2, Ba(OH)2 castro bear

Gas effusion equation

M: molar mass

mechanical advantage of a ramp formula

MA = output force/input force MA = hypotenuse / height ir on ramp

Mass spectrometry, IR spectroscopy, Gas chromatography, UV-Vis spectroscopy, H-NMR Spectroscopy A. used to determine if specific functional group are present in a given molecule B. This technique detects conjugated pi systems and metal-to-ligand charge transfer species, both of which exist here. The appearance of a peak corresponding to the charge transfer component will appear C. molecules flow through a coated tube separates compounds by their size, weight, and chemical reactivity with the coating of the tube or column D. type of absorption spectroscopy which tells us several things the # of sets of peaks in the spectrum and tells the splitting pattern of each set of peaks (how many protons are interacting with the protons in that set) E. an experimental method of determining the precise mass and charge ratio

Mass spectrometry: E. an experimental method of determining the precise mass and charge ratio IR spectroscopy: A. used to determine if specific functional group are present in a given molecule Gas chromatography: C. molecules flow through a coated tube separates compounds by their size, weight, and chemical reactivity with the coating of the tube or column Hint: this molecules are always gas molecules UV-Vis spectroscopy: B. This technique detects conjugated pi systems and metal-to-ligand charge transfer species, both of which exist here. The appearance of a peak corresponding to the charge transfer component will appear H-NMR Spectroscopy: D. type of absorption spectroscopy which tells us several things the # of sets of peaks in the spectrum and tells the splitting pattern of each set of peaks (how many protons are interacting with the protons in that set)

match each with its location on a benzene ring with a group meta, para, ortho A. one carbon away from group B. two carbons away from group C. Opposite sides, 3 carbons away, from group

Meta: B. two carbons away from group Para: C. Opposite sides, 3 carbons away, from group Ortho: A. one carbon away from group

moles of metal from electroplating equation

Mol metal = It/nF I-current t-time n-moles of e- transferred F- constant = 1E5

How do you make alcohols more acidic and more basic?

More acidic: add electron withdrawing substituents Ex: add electronegative atom, like Cl on a carbon close to the OH makes the conjugate base of the acid more stable and therefore the acid more acidic. The more Cl the more stable the CB is. Less acidic: add electron donating substituents Ex: CH3

draw an amine

N with three attachments primary- 1 R group secondary- 2 R groups tertiary- 3 R groups

Pascals are equal to what?

N/m^2

strong bases organic chem

NH- H- tert butoxide -CN

why is it 13C NMR and not 12C

NMR will only detect spin when when protons are different from neutrons 12C has the same protons to neutrons

write out NAG SAG and what each letter means what is this used to tell us?

Nitrates Acetates Group 1 Sulfates Ammonium Group 17 tells us what molecules are always soluble exceptions are insoluble

On the topic of angle strain, what angle does carbon normally want to exhibit? what happens if it does not exhibit this angle? what does this have to do with the chair conformation?

Normally wants a tetrahedral angle of 109.5 if it doesn't exhibit this angle it will be unstable the chair conformation is a 111 angle so it adapts this shape

acid catalyzed Aldol Condensation

O is protonated creating +OH alpha C is deprotonated and double bond is turned into lone pair on O creating enol a second aldehyde is protonated enol attacks aldehyde at carboxyl C enole +OH is depronated making the aldol (aldehyde + alcohol)

HB + H2O --> ?

OH- + HB+ makes your OH as seen in basic solutions

doublet on NMR

One adjacent hydrogens Ex: C2HC-C-H3

do current and electrons travel in the same or the opposite direction

Opposite

acid and bases ph scale strong base weak base neutral weak acid strong acid

PH scale indicates the concentration of H+ ions in a solution 14-10: strong base 10-7: weak base 7: neutral 6-4: weak acid 3-0: strong acid

What is the exception to NAG SAG

PMS Castro Bear

What does PMS Castro Bear stand for regarding solubility

PMS Castro Bear is the exception to NAG SAG Pb+ (lead) Mercury (Hg) Silver (Ag) Ca+2 Sr+2 Ba+2 these are not always soluble

what acid is this

Palmitoleic acid hydrophobic part (its long carbon chain) and a polar carboxylic acid

what class of molecule is this

Prostaglandins derived from arachidonic acid and have 20 carbon atoms and one 5-carbon ring

Aldehydes formula and suffix

R-CHO Suffix: -al or carbaldehyde

Amides formula and suffix

R-CO-NR2 Suffix: -amide

Ketones formula and suffix

R-CO-R Suffix: -one

Acid Halide formula and suffix

R-CO-X Suffix: -oyl halide

ester formula and suffix

R-COO-R Suffix: -oate

carboxylic acid formula and suffix

R-COOH Suffix: -oic acid

what products are made when an amide and water and catalyzed with H2SO4 and heat RCOONR2 + H3O ---> ?

RCOOH and NHR2 this is called amide hydrolysis

what products are made when an ester and a primary alcohol and catalyzed with H2SO4 and heat RCOOR + R'OH ---> ?

RCOOR' and ROH the Rs attached to the oxygens switched this is called transesterification

half life equation

Remaining atom = Original atoms * (1/2)^# half lives that has passed

draw a thioether

S with 2 R groups

what causes an SN1 reaction to occur rather than an SN2

SN1 reactions occur when the carbocation formed is tertiary or secondary (more stable)

SN1 vs SN2

SN2 bimolecular and rate depends on electrophile and nucleophile concentrations SN1 unimolecular and rate depends only on electrophile

Sulfate reacts with methane through the following reaction: CH4 (g) + SO4 (aq) → H2O (l) + HS- (aq) + HCO3- (aq) In a college biology experiment, 50 mg of methane and 60 mg of sulfate are mixed in a closed container. What is the limiting reagent and how much excess reagent remains when that species has been exhausted? A. CH4, 55mg of SO4 B. CH4, 50mg of SO4 C. SO4, 40mg of CH4 D. SO4, 10mg of CH4

SO4, 40mg of CH4 Methane: 0.05g x 1mol/12g+(4)(1g) = 0.003 mol Sulfate: 0.06g x 1mol/32g+(4)(16g) = 0.0006 mol sulfate is smaller so it is the limiting reagent to find excess CH4 0.003mol - 0.0006mol = 0.00024mol CH4 x 12g+(4)(1g)/1mol = 0.04g or 40mg

How does salt affect freezing point?

Salt in water lowers the freezing point of water.

STP temperature and pressure

Temp: 273K (0ºC) Pressure: 1 ATM

ionization energy

The amount of energy required to remove an electron from an atom easiest and lowest to remove first electron in alkali metals

naming ionic compounds

The name of the metal comes first, followed by the name of the nonmetal, no suffix, changing the nonmetal's ending to "ide".

absorption spectrum

The range of a pigment's ability to absorb various wavelengths of light. red has the highest wavelength of near 700 violet has the lowest near 400

why is a tertiary alcohol a better substrate for an SN1 reaction than a primary alcohol

The tertiary alcohol forms a cation tertiary carbocation whereas the primary alcohol form an unstable primary carbocation

How are absorbance and transmittance related?

They are inversely related

Match each test with its definition Tollens test, benedict's test, fehlings test A. copper sulfate(blue) ions oxidize an aldose sugar to produce a copper oxide (red) which can be seen in the solution B. Oxygen is used to oxidize a ketose sugar to produce a blue precipitate to ensure that the sugar is in the solution C. silver ions oxidize an aldose sugar to produce solid silver which can be seen in the solution D. copper sulfate is used to estimate the amount of blood glucose levels

Tollens test: C. silver ions oxidize and aldose sugar to produce solid silver which can be seen in the solution Hint: used to distinguish between aldehydes and ketones Benedict's test: A. copper sulfate(blue) ions oxidize an aldose sugar to produce a copper oxide (red) which can be seen in the solution Hint: tests for glucose Fehlings test: D. copper sulfate is used to estimate the amount of blood glucose levels

match each type of strain with its definition torsional strain, steric strain, angle strain A. results when bond angles deviate from their ideal values by being stretched or compressed B. the interference between two bulky groups that are so close together that their electron clouds experience a repulsion C. Increased energy that results when molecules assume eclipsed or gauche staggered conformations

Torsional strain: C. Increased energy that results when molecules assume eclipsed or gauche staggered conformations Steric strain: B. the interference between two bulky groups that are so close together that their electron clouds experience a repulsion Angle strain: A. results when bond angles deviate from their ideal values by being stretched or compressed

what class of molecule is this

Triacyl glycerols fatty esters made up of long hydrocarbon chains

True or false: Carbonic acid (H2CO3) is a polyprotic molecule.

True

True or false: Hydroquinone reacts with an oxidizing reagent to form quinone

True

True or false: a homogenous catalyst is a catalyst that is in the same phase(liquid, gas) as all the reactants and products in a reaction system

True

True or false: fractional distillation is better for separating two solutions with very close boiling points, whereas simple distillation is used to separate solutions with more diverse boiling points

True

True or false: in most chemical reactions, reaction rate is dependent on reactant concentration

True

True or false: vapor pressure is the pressure exerted by a vapor over a liquid

True

True or false: you can have a pH<0 or pH>14

True

True or false: ∆G is 0 at equilibrium

True

true or false: a chiral center needs 4 different substituents attached

True

true or false: absolute configuration describes the exact spatial arrangement of groups of atoms independent of other molecules based on their atomic number: R and S

True

true or false: you can have a negative pH

True this would be a very acidic solution

true or false: Homotropic regulation is when the substrate of an enzyme also regulates its activity

True (Ex: O2 is a homotropic allosteric modulator of hemoglobin) cooperativity

True or false: sugar cyclization is a reaction occurring in linear carbohydrates where the hydroxyl on the 4th or 5th carbon attacks the carboxylic group forming either hemiacetal or hemiketal structures

True hemiacetal or hemiketal structures are made based on the hydroxyl group attacking an aldehyde or a ketone it also produces 2 different anomers with the OH on carbon 1 either pointing up or down depending on the side of attack, these are stereoisomers

True or false: forming bonds releases energy

True this is because the bonded atom is more stable and therefore has a lower energy meaning the extra energy was released into the environment

True or false: geometric isomers(cis-trans isomer) only occur when 2 of the same molecules are on opposite sides

True there needs to be and X and another X on the other side, both need to be the highest priority substituents if there are 4 different ones, we have E and Z

True or false: When considering the components of the blood's buffer system, carbonic acid is the weak acid and bicarbonate is its conjugate base.

True: The blood buffer system is composed of a weak acid and its conjugate base. Carbon dioxide in the blood is converted to carbonic acid, which readily dissociates to form bicarbonate.

True or false: nuclear fusion requires energy but also releases energy

True: the combination of the nuclei requires energy but once bound the binding energy of the atom is released to create energy

Ohm's law equation

V = IR

viscosity vs surface tension A. A liquid's resistance to flowing B. A measure of how difficult it is to stretch or break the surface of a liquid

Viscosity: A. A liquid's resistance to flowing Surface tension: B. A measure of how difficult it is to stretch or break the surface of a liquid

P∆V =

W

Michael addition reaction

We're *forming a C-C bond*, and we're *breaking a C-C Pi bond*. produces a 1,5- dicarbonyl C=O on Cs 5 spaces away After we do the addition, we form an enolate - so adding acid gives us the neutral product

when will E1 reactions occur over E2

Weak base will be an E1 reaction Acidic environment will favor an E1 reaction Strong base will be E2 Strong basic environment will be E2

combustion reaction

When a substance combines with oxygen (O2) to produce CO2 and H2O.

____-ite

XO2 1- -ite means less oxygens normally 2 oxygens

___-ate

XO3 1- more oxygen atoms, normally 3

per-___-ate

XO4- one more oxygen than -ate

Can liquid-liquid extraction separate solutes that are both neutral resulting in them being found in the organic phase

Yes, if they have other differing properties that allows them to be separated into different liquids Ex: acid and base groups can help them be separated if the pH is altered enough for one to have a charge

Match the term on the left to the units on the right. Zero order, First order, second order, third order A. 1/(M•s) B. 1/(M^2•s) C. M/s D. 1/s

Zero order: C. M/s First order: D. 1/s second order: A. 1/(M•s) third order: B. 1/(M^2•s)

Cu electron configuration

[Ar] 4s1 3d10 one electron was removed from 4s1 because a single electron in an s orbital is much more stable than only 9 electrons in a d orbital orbitals are happy when completely full or half full, ex Cr for Ni all 4s electrons are given to 3d

specific rotation formula

a = observed rotation / concentration of solution(g/mL) x length(dm)

Describe where solid, liquid and gas are on a phase diagram

a graph showing the conditions at which a substance exists as a solid, liquid, or vapor

Van der Waal Equation

a mathematical expression that describes the behavior of non ideal gases a- molecular attraction forces b- size of particles

Positron

a particle with the mass of an electron but a positive charge

What happens when an electron jumps from shell n = 1 to shell n = 2

a photon is absorbed and electron moves to an excited state

NADPH + H + O2 + R-H → NADP + H2O + R-OH in this equation NADPH is a: A. an oxidizing agent. B. a reducing agent. C. a catalyst. D. an electrophile.

a reducing agent Although NADPH is itself oxidized, its role is to transfer these electrons to O2 to reduce it to H2O. Thus, NADPH is not an oxidizing agent; it is a reducing agent.

thin layer chromatography

a separation technique that involves the separation of small molecules as they move through a silica gel

explain the process of titration. Use the word bank below known solution, unknown solution, buret, dripped, color change, equivalence point, volume, concentration

a solution with a known concentration (known solution) is in the buret which is dripped into an unknown solution until a color change occurs. This is the equivalence point. The change in volume of the known solution is used to find the concentration of the unknown solution.

Buffers consist of A. a weak acid and its conjugate base B. any strong acid and a weak base C. any weak acid and a strong base D. a strong acid and its conjugate strong base

a weak acid and its conjugate base or visa versa

Lewis acid/base

acid: electron pair acceptor base: electron pair donor

mechanism of acyl substitution

addition-elimination process

adhesive vs cohesive forces

adhesive: the force between two different substances that produces capillary action Ex: water and its container cohesive: the force between molecules of the same substance Ex:

A hydrolysis reaction turns an ester into a

alcohol and carboxylic acid

Are aldehydes or ketones more acidic

aldehydes are more acidic because it only has one electron donating group (CH3) instead of two, making it more stable

base catalyzed keto-enol tautomerization

alpha hydrogen is removed via OH and bond is moved between alpha carbon and carbonyl C C=O double bond is pushed up as a lone pair on the O becoming - with 3 lone pairs O attacks H on water with one lone pair making OH

match each type of radiation with the term that describes it, multiple definitions for each type alpha particles, beta particles, gamma rays A. positively charged B. negatively charged C. Neutral D. small E. large F. most penetrating G. medium penetrating H. least penetrating

alpha particles: A. positively charged E. large H. least penetrating beta particles: B. negatively charged D. small G. medium penetrating gamma rays: C. Neutral F. most penetrating

do amides or carboxylic acids have higher boiling points

amide because of its hydrogen bonding and resonance charge distribution

what group is pictured

amine RNH2, R2NH, R3N

amorphous solids vs crystalline solids

amorphous solids: the particles are not arranged in a regular pattern crystalline solids: particles are arranged in repeating patterns based on bonds and intermolecular forces

Do cations or anions have a larger radius?

anions because they gain valence electrons and the other electrons repel

what molecules can diffuse through a cell membrane

any sized nonpolar molecules, small polar uncharged molecules, gases

can molarity and molality be used interchangebly at times?

at low solute concentrations in aqueous solvents they can be but not at high solute concentrations

effective nuclear charge (Zeff)

attractive force between the atoms positively charged nucleus and the negatively charged valence electrons increases with more protons (right of table) increases with less electron shells (up the table)

why does a strong acid have a weak conjugate base

because Ka * Kb = Kw as Ka increase Kb decreases Ka represents the acidity of an acid Kb would represent the basicness of the acids conjugate base and visa versa

Why is a phenol more acidic than a regular alcohol?

because it has resonance so its conjugate base is much more stable making its acid more acidic

which is more stable, the alpha or the beta anomer of a carbohydrate BONUS: why

beta because when in the chair position, the OH is equatorial

beta minus decay, beta plus decay A. loses positive charged positron B. loses - charged e

beta minus decay: B. loses - charged e beta plus decay: A. loses positive charged positron

bond dissociation energy equation

bonds broken - bonds formed pure elements are incorporated

coordinate covalent bond

both bonds come from the same atom in the picture both the electrons to create the bond come from O HCl would not be coordinate because one electron comes from H and one from Cl to create the bond

Ecell

cell potential

The pressure of a gas is caused by

collisions of molecules with the walls of the container Pressure is AKA Force/area of container Pascal = N/m^2

aryl amine definition draw it

compounds in which nitrogen is bonded to an sp2 hybridized carbon of an aromatic ring

Alkyl amines definition draw it

compounds in which nitrogen is bound to an sp3-hybridized carbon

SN2 reaction mechanism

concerted 1 step no carbocation formed backside attack

E2 mechanism

concerted in one step Bronsted lowrey base removes H leaving lone pair with makes the most substituted double bond kicking off the lone pair

what group of molecules can be easily detected by UV spectroscopy for their ability to absorb UV light

conjugated molecules

As atmospheric pressure decreases, boiling point ___________ BONUS: why?

decreases Lowering the pressure lowers the boiling point because the molecules need less speed to escape. The low atmospheric pressure on high mountains lowers the boiling point to such an extent that water cannot get hot enough to boil eggs satisfactorily.

are glycosidic bonds formed through hydration or dehydration reactions

dehydration reactions

pKa < pH

deprotonated

draw imine

double bond between carbon and nitrogen with nitrogen attached to an R group

Draw the structure of an enol

double bond has been removed between carboxyl carbon and O and now sit between carboxyl carbon and alpha C Enol is a combo of the suffix -ene(C=C) and -ol (OH)

Which of the following processes describe a phase change that leads to increased entropy? Select all that apply. A. vaporization B. condensation C. deposition D. melting

entropy is (S) which is the disorder of a system As disorder increases, S increases A. vaporization D. melting

acetic acid

ethanoic acid

True or false: NMR spectroscopy measures the energy state of an atom in a magnetic field

false: it measures how often the atom switches between a high energy state and a low energy state in a magnetic field (this is called resonance)

true or false: methionine can form disulfide bonds

false: only cysteine

True or false: taking away the common ion from the products side causes the reaction to shift towards the product side the the react and the reactant becomes less soluble

false: the reactant would want to make more product so it would become more soluble

True or false: 1,4 beta glycosidic bonds between carbohydrates form from two carbons with the same substituent configuration

false: these form from C 1 and C4 with different configurations

true or false: the equation, Ma x Va = Mb x Vb, can be used at anytime during an acid base titration to determine the concentration of the unknown solution

false: this can only be used when the titration reaches its equivalence point. That is when these two values will be equal. We do the whole titration to get to this point specifically to use this equation.

True or false: when finding the absolute configuration of a stereocenter the lowest priority molecule (normally a H) needs to be pointing out towards you

false: when finding the absolute configuration of a stereocenter the lowest priority molecule (normally a H) needs to be pointing in away from you if it is not this way, then once you find your configuration (R or S) switch it

SN1 mechanism

first reaction is slow the second reaction is fast step 1: leaving group leaves and generates a carbocation may need to make a good leaving group before this occurs step 2: nucleophile attacks may need to be deprotonated after attack

how do you find Keq for the reverse reaction

flip the Keq equation for the forwards reaction

match each definition to its term galvanic cell, electrolytic cell A. cathode is + B. cathode is - C. anode is + D. anode is -

galvanic cell: A. cathode is + D. anode is - electrolytic cell: B. cathode is - C. anode is + still An Ox Red Cat

galvanic cell, Daniell cell, concentration cell, electrolytic cells, electroplating A. type of galvanic cell that uses a salt bridge to ensure the reaction continues instead of halting after charges have been swaped B. spontaneous is used to generate a positive potential difference that drives current C. nonspontaneous reduction of metal ions in a solution D. a galvanic cell in which both compartments contain the same components, but at different concentrations E. electrical energy is used to drive a nonspontaneous redox reaction

galvanic cell: B. spontaneous is used to generate a positive potential difference that drives current Daniell cell: A. type of galvanic cell that uses a salt bridge to ensure the reaction continues instead of halting after charges have been swaped concentration cell: D. a galvanic cell in which both compartments contain the same components, but at different concentrations electrolytic cells: E. electrical energy is used to drive a nonspontaneous redox reaction electroplating: C. nonspontaneous reduction of metal ions in a solution

alkene suffix and definition

has at least one carbon carbon double bond -ene

Match each term with its definition hemiacetal and hemiketal A. Have an OH, OR, H, and R group attached to the same carbon B. Have an OH, OR, and two R groups attached to the same carbon

hemiacetal: A. Have an OH, OR, H, and R group attached to the same carbon aldehyde + alcohol hemiketal: B. Have an OH, OR, and two R groups attached to the same carbon ketone + alcohol

what causes certain substituents to have higher priority over others

higher oxidation state the greater the degree of oxidation (more CO bonds) the more priority a substituent has the molecule will be numbered so the highest priority group is the lowest and will have the suffix of this group

phthalic anhydride

hydrolyzes to phthalamic acid in water

The pH of a substance is a measure of its

hydronium ion concentration

Keq equation are there any exceptions

if one of the products or reactants are not gas or aqueous then they are not to be inserted into the equation reaction must be at equilibrium to use this equation

if we add acid to a buffered equation, based off this equation pH = pKa + log [A-]/[HA] what happens to the concentration of A- and HA? how does this effect pH?

if we add acid to a solution, more conjugate base A- will be consumed to neutralize the acid. Therefore, the [A-] will decrease since we are adding acid, the concentration of [HA] will increase this makes the log value more negative and therefore will lower pH obviously adding more acid will lower the pH in general

Reduction is an increase in C-___ bonds and a decrease in C-___ bonds

increase in C-H bonds Decrease in C-O bonds or bonds with another electronegative element (N, S)

how to speed up an SN1 reaction

increase with a good leaving group. polar protic solvents helps stabilize the transition state and carbocation intermediate

describe how Infrared spectroscopy works

involves the casting of infrared light through a molecular sample. The energy from the light then causes covalent bonds within the molecule to vibrate. These bonds can vibrate by stretching and bending. As the light passes through the sample, a spectrometer measures the percentage of light that passes through the other end. This percentage is expressed as the percent transmittance of light over a range of frequencies. The frequencies at which the light is absorbed depend on the types of functional groups present in the sample. Thus, IR spectroscopy can identify specific functional groups and types of bonds in a compound.

the common ion effect

is a decrease in the solubility of an ionic compound caused by the addition of a common ion that is already found in the products when adding common ion reaction is shifted to the reactants and the original reactant is less soluble less soluble because we want more reactant and if it's less soluble then it wont dissociate as much to form product

what substituent is this?

isobutyl

what is this molecules name

isopropyl

Does LAH reduce carboxylic acid, aldehydes, or both into primary alcohols, making it a strong reducing agent

it reduces both so it is a strong reducing agent

what is normally more stable, the enol or the ketone

ketone

A lower activation energy of a reaction is this thermodynamically favored or kinetically favored?

kinetically favored the red line in the graph, kinetically favored reactions want smaller activation energy and don't care that the product is higher in energy

Kinetics vs. Thermodynamics

kinetics- rate, intermediate, *activation energy*, catalyst faster product thermodynamics- stability, equilibrium, spontaneity, entropy, enthalpy, *free energy* more stable product for example, free energy of ion transport does not tell you about the kinetics of a channel protein

Backside attack of SN2

leads to inversion. Only one diastereomer is produced S would become R and vis versa the nucleophile attacks the molecule as far away from the leaving group

a molecule with an OH will move _________ (more/less) on a silica chromatography compared to a nonpolar molecule

less, silica is polar and therefore will force the polar molecule to move less and will not affect the nonpolar compound

does a high Ka mean a high or low pKa

low pKa they are inversely proportional same with Kb and pKb these are also inversely proportional

in NMR spectroscopy when atoms spin is aligned with the magnetic field do they have a lower or higher energy state

lower energy state think, if you go with the flow, you're not exerting so much energy to work against

is a higher temp or lower temp better for dissolving gas in a liquid solution

lower temp, the gas particles have less energy and therefore are more likely to stay in the solution rather than reeveporate our

formic acid

methanoic acid

Normality (N) equation and use

molarity x n n- number of acidic protons or basic hydroxide ions per molecule or equivalents/L of solution this equation is used when we dissolve ionizable compounds (NaCl) in solution and we want to find the number of Na+ ions or Cl- ions after its dissociated

vapor pressure of solution is equal to

mole fraction of solvent x vapor pressure of pure solvent mole fraction of solvent: the amount of solvent particles relative to the total particles of a solution will decrease as more solute is added

Molality equation (this is not M)

moles of solute/kg of solvent (only the liquid solvent, not solution)

Molarity equation

moles of solute/liters of solution

Prefix's for covalent compounds, 1-10

mono, di, tri, tetra, penta, hexa, hepta, octa, nona, deca

If conjugate base is stable is it more acidic or basic

more acidic

What happens when an electron jumps from shell n = 2 to shell n = 1

moves to a more stable state and a photon is emitted

quantum numbers

n- energy level of electron l- angular momentum, subshell of electron, s=0, p=1, d=2, f=3 ml- spatial orientation of electron, -2, -1, 0, 1, 2 ect. ms- spin, +1/2 or -1/2

which is more polar 2-butanone or n-butanol

n-butanol 2-butanone: ketone, pka = 22 n-butanol: alchol, pka = 17 lower pka means more acidic and more polar

are double displacement (metathesis) reactions redox reactions?

no you can find the oxidation state of each atom but it does not change in double displacement reactions therefore is not a redox reaction

does adding salt to pasta water make it boil faster?

no adding solute to solvent increases boiling point, meaning we need more heat to make it boil it may make your food cook faster tho

singlet on NMR

no adjacent hydrogens Ex: C3C-C-H3

does O2 have a ∆Hf

no because it is a pure element in its standard state so it will not have a ∆Hf and can be excluded from ∆Hrxn equations

Pauli Exclusion Principle

no two electrons in the same atom can have the same set of four quantum numbers

is radioactive Decay reversible

no, one the element has become another, there is no decay to go backwards

what causes SN2 reaction to occur more often than SN1

nucleophile must be strong and small (to eliminate steric strain) substrate should also be primary or methyl group to avoid steric strain

Substitution mechanism

nucleophilic attacks at carbon and good leaving group (Cl, Br, F, H2O) leaves

Normality (REDOX)

number of electrons given up or absorbed by a chemical per mole of chemical n1M1V1 = n2M2V2 n-moles M-molarity V-volume

if a compound has both an OH and NH2 in it, will the suffix be -ol or -anamine

ol because thats the more important group

Henderson-Hasselbalch equation

pH = pKa + log [A-]/[HA] This equation relates pH, pKa and concentrations of the buffer pH: pH of buffer solution pKa: pKa of weak acid [X]: can either be moles or concentration of the chemical, needs to be the same for both tho

PPM stands for

parts per million

which has a high boiling point, butanol or pentanol

pentanol both have the same group that hydrogen bonds so the only difference is an extra carbon Large molecules have more electrons and nuclei that create van der Waals attractive forces

When the acid and conjugate base concentrations are equal, the pH of a buffer =

pka of the acid

naming molecular compounds

prefix + 1st element + prefix + base name of 2nd + -ide prefix not given if only one of first element must give roman numeral of transition metal

pKa > pH

protonated

specific heat equation used to find amount of heat lost from a substance

q = mcΔT Q = heat (calories NOT Calories) m = mass c = specific heat constant (1 for H2O) T = Temp (K)

electronic geometry

refers to the position of all electrons in a molecule, whether bonding or nonbonding lone pairs do not change the shape of the molecule and are kind of counted as a bond H2O has a tetrahedral shape

Endomembrane system

regulates protein traffic and performs metabolic functions in the cell the golgi apparatus

how do you separate enantiomers

resolving agent that reacts with only one of the enantiomers

Why do we put salt on sidewalks when it snows?

salt lowers the boiling point of water, for the sidewalks to freeze, it needs to be much colder than normal

draw a thioester

same idea as an ester but with S instead of O

what substituent is this?

sec-butyl

in gas-liquid chromatography, smaller compounds elude A. first B. last

smaller compounds with lower boiling points

What chemical property is NAG SAG used for

solubility rules these NAG SAG compounds are always soluble

hybridization XeF4

sp3d2 XeF4 has two lone pairs (d2) and four bonds (sp3) around the central xenon atom.

chromate

special like S, it has an extra O

sulfate

special, has 1 extra O

Frequency of light equation

speed of light(c)/wavelength(λ)

spontaneous and nonspontaneous reaction G, Keq, and E cell

spontaneous: G<0 Keq>1 E cell>0, always opposite of G nonspontaneous G>0 Keq<1 E cell<0

what class of molecules is this

steroid hormones or cholesterol have a fused 4 ring system

finding reaction direction at galvanic cell

take reduction potential at cathode + the oxidation potential at the anode

the equation q = mcΔT tell us what and what does each variable mean

tell us how much temperature changes from the addition of heat q: heat (kJ) m: mass (kg) c: specific heat (kJ/kg x K or J/g x C), given T: temp(K)

the equation q = mΔHx tell us what and what does each variable mean

tells is the heat needed to transition into the next phase temperature does not change during this stage q: heat (kJ) m: mass (kg) Hx: is heat of (x, this is the phase change. Can be any of the 6. Ex: fusion(f), vaporization(v)) for that substance

match each term with its definition the anode of an electrochemical cell, cathode of an electrochemical cell A. where reduction occurs B. where oxidation occurs

the anode of an electrochemical cell: B. where oxidation occurs cathode of an electrochemical cell: A. where reduction occurs An Ox Red Cat

molecular geometry

the arrangement of bonded atoms doesnt include lone pairs Ex: H2O is bent with only 2 bonds

How is the phase diagram for water different from most substances?and why?

the boundary line between solid and liquid is slanted to the left where on most phase diagrams it is to the right this is because water is less dense in its solid form than in its liquid form

Archimedes' principle

the buoyant force on an object is equal to the weight of the fluid displaced by the object

Venturi effect

the decrease in pressure that occurs when a fluid flows into a constricted region of a pipe

calorie definition

the energy needed to raise the temp of one gram of liquid water by one degree celsius specific heat but for water

specific heat capacity definition and equation

the energy required to raise the temperature of one gram of a substance by one degree Celsius

how does effective nuclear charge (Zeff) affect atomic radius

the more attraction between the nucleus and the valence electrons, the smaller the radius because it pulls the electrons closers so effective nuclear charge (Zeff) and atomic radius are opposite

During a liquid-liquid extraction what solution is removed first

the more dense solution is removed first, then the less dense is removed second

what happens to the bond angle of atoms with lone pairs

the more lone pairs the smaller the bond angle

Why don't the protons in the nucleus of an atom repel each other?

the nuclear force of the nucleus is stronger than the electromagnetic force pushing the protons apart

bond order

the number of shared electron pairs between two atoms Ex: N=N has a bond order of 2

conjugate base

the particle that remains when an acid has donated a hydrogen ion

neutralization reaction

the reaction of an acid and a base to form a neutral solution of water and a salt

if we add more reactant into a reaction at equilibrium what will occur

the reaction will shift to the right and create more product to return to equilibrium

Aufbau Principle

the rule that electrons occupy the orbitals of lowest energy first follow the exact levels of the periodic table

reactant quotient (Q)

the same equation as Keq but this can be used anytime during the reaction unlike Keq that can only be used during equilibrium must be compared to Keq to have relevance

Period of a time-varying signal

the shortest repetition time in the picture, this would be 1000ns

what occurs when you try to add more solute to a saturated solvent

the solute will precipitate out

how is the IUPAC name of ester different than other molecules

their names involve two different side chains, one on one side of the oxygens and the other on the other -oate is given to the chain by the double bonded oxygen -ly is given to the chain bounded to the oxygen

A lower energy intermediate is formed is this thermodynamically favored or kinetically favored?

thermodynamically favored This would create a more exothermic reaction which is more thermodynamically favored the blue line in the graph, thermodynamically favored reactions are okay with a larger activation energy as long as the product is lower in energy

draw a thiol

thi - sulfur ol- alcohol (the O has been replaced with the S in this instance) S with 1 H and 1 R group

During a liquid-liquid extraction, why do you have to shake the solutions vigorously?

this is what allows the solutions to separate so they can be clearly poured into different containers

draw the half equivalence point on a titration graph

this is where pH = pKa concentration of original weak acid is equal to the concentration of its conjugate base

in thin layer chromatography which travels farther, polar or nonpolar compounds

this test is a separation technique that involves the separation of small molecules as they move through a silica gel nonpolar compounds travel farther because they are dissolved in the the mobile phase that the paper is placed in the polar compounds are more attracted to the silica gel on the paper and don't move as far

what occurs if a reaction is exothermic and we increase heat A + B -> C + Heat

to tell what a temp change will do to a reaction we must view heat as a reactant or product if heat is increased then this makes the "concentration" of our product increase meaning the equilibrium will shift to the left to increase reactants

Are trans or cis isomers more stable?

trans

True or false: Ka = Keq for an acid dissociation reaction

true

True or false: the anomeric carbon is the carboxyl carbon that is attacked by the OH when forming a cyclic carbohydrate

true

true or false: for keto-enol tautomerization, there needs to be a alpha hydrogen

true

True or false: Aldehydes and ketones can act as both nucleophiles and electrophiles

true they can both attack or be attacked

True or false: the equivalence point is the point at which the acid and base used in a titration are neutralized

true the moles of acid = moles of base

True or false: an achiral molecule is a rotated molecule that can be superimposed on its mirror image

true has 2 of the same groups or only 3 bonded atoms or is a meso compound

true or false: dehydration reactions involve carbocation intermediates

true: carbocation is formed when OH is hydrolyzed to H2O and leaves forming a + C, then H is removed the the bond attaching H creates a double bond

what does PBr3 do to carboxylic acids and to alcohols

turn OH into Br

What does SOCl2 do to a carboxylic acid?

turn OH into Cl

triplet on NMR

two adjacent hydrogens Ex: in pic, red H is the only being measured with 2 H on the carbon next to it, causing the triplet

geometric isomers(cis-trans isomer) vs conformational isomer

type of stereoisomer that has the same molecular formula but differs in the spatial arrangements of their atoms. cannot be rotated like a conformation isomer because it has a double bond

obligate parasites

unable to grow outside of a living host

work efficiency formula

useful work done/ total energy input

VESPR theory

valence shell electron pair repulsion theory, because electron pairs repel, molecules adjust their shapes so that valence electrons pairs are as far apart as possible

formula with planks con, mass, velocity, and wavelength

wavelength = h/mv h-planks m-mass v-velocity

Can an endothermic (+ΔH) reaction be spontaneous (-ΔG)?

yes, although less common if the temp or entropy is high enough it will outweigh the positive ΔH ΔG = ΔH - TΔS

is pyrrole aromatic?

yes, there are 4 pi bonds in the 2 double bonds and there are 2 more pi bonds in the lone pair on the N this makes 6 e total 4n + 2 = 6 n = 1, a whole number

one mole of HBr and 2 moles of NH3 react, is this a buffer question?

yes, this is because one mole of HBr will react with exactly one mole of NH3 to produce a neutralized solution, except 1 mole of NH3 remains causing the solution to be basic NH3 gains the H from HBr making it NH4+ the conjugate acid. the weak base ammonia is titrated to form its conjugate acid making it a buffer question

ΔG and Keq equation

ΔG = -RTln(Keq)

Gibbs free energy equation

ΔG = ΔH - TΔS H: enthalpy S: entropy G: gibbs free energy T: temp (K)

Enthalpy equation

∆H = ∆E + P∆V = Q H: enthalpy (J) E: internal energy P: pressure V: volume Q: heat lost or gained by the system

∆Hrxn= math not conceptual

∑∆Hreactant bonds broken + ∑∆Hproduct bonds formed


Related study sets

Church History Study Guide Unit 4-3

View Set

2.1 Examining Relationships Introduction: Role Type Clasification

View Set

EPS 601 Chapter 1 (Introduction and overview, study guide)

View Set

Cognitive Psychology Ch. 4 and 5

View Set